SlideShare uma empresa Scribd logo
1 de 125
Análise Combinatória

Fatorial de um número:

  n!=n.(n-1).(n-2)...3.2.1

Definições especiais:
                                   0!=1
                                   1!=1


                            100!+101!
1) Calcule o valor da expressão       .
                               99!
100!+101! 100.99!+101.100.99!
         =                    = 100 + 101.100 = 100 + 10100 = 10200
   99!            99!

                      ( x + 1)!
2) Resolva a equação             = 56.
                      ( x − 1)!
( x + 1)!        ( x + 1)( x)( x − 1)!
          = 56 ⇒                       = 56 ⇒ ( x + 1)( x) = 56 ⇒ x 2 + x = 56 ⇒
( x − 1)!              ( x − 1)!
                                − 1 ± 225            − 1 ± 15     x = 7
  ⇒ x 2 + x − 56 = 0 ⇒ x =                     ⇒ x=           ⇒
                                      2                 2         x = -8
Resposta : x = 7, pois não existe fatorial de um número negativo.


3) Quatro times de futebol (Grêmio, Santos, São Paulo e Flamengo) disputam o torneio dos
campeões do mundo. Quantas são as possibilidades para os três primeiros lugares?
R : Existem 4 possibilidades para o 1º lugar, sobrando 3 possibilidades para o 2º lugar e 2
possibilidades para o 3º lugar → 4.3.2 = 24 possibilidades.

Arranjo simples:

                       n!
        An , p =
                    (n − p)!


               A6, 2 + A4,3 − A5, 2
4) Calcule                            .
                    A9, 2 + A8,1
                             6!         4!        5!
                                   +        −
 A6, 2 + A4,3 − A5, 2     (6 − 2)! (4 − 3)! (5 − 2)! 30 + 24 − 20 34 17
                        =                            =           =   =
     A9, 2 + A8,1                  9!       8!          72 + 8     80 40
                                        +
                                (9 − 2)! (8 − 1)!




                                                                                              1
5) Quantos números de 3 algarismos distintos podemos formar com o algarismos do
sistema decimal (0,1,2,3,4,5,6,7,8,9) sem os repetir, de modo que :
a) COMECEM COM 1.
   R : O número pode possuir três algarismos, sendo que para o primeiro existe apenas 1
possibilidade (1) e para os outros dois ainda existem 9 números disponíveis :
                 9!    9! 9.8.7!
   1. A9, 2 =         = =        = 9.8 = 72 números.
              (9 − 2)! 7!    7!

b) COMECEM COM 2 E TERMINEM COM 5.
   R : Para o primeiro algarismo existe apenas 1 possibilidade (2), e para o terceiro também
existe apenas 1 possibilidade (5). Para o segundo ainda existem 8 possibilidades :
                 8!    8! 8.7!
  1.1. A8,1 =         = =        = 8 números.
              (8 − 1)! 7! 7!


c) SEJAM DIVISÍVEIS POR 5.
   R : Para um número ser divisível 5, ele deve terminar com 0 ou com 5. Primeiramente
vamos calcular o número de divisíveis por 5 que terminam com 0 :
→ Para o terceiro algarismo existe apenas 1 possibilidade (0), e para os dois primeiros ainda
existem 9 números disponíveis. Portanto o número de divisíveis por 5 que terminam com 0 é :
                   9!       9! 9.8.7!
   1. A9, 2 =            = =           = 9.8 = 72 números.
                (9 − 2)! 7!        7!
→ Agora calculamos quantos divisíveis por 5 terminam com 5 : para o terceiro algarismo
existe apenas uma possibilidade (5). Para o primeiro algarismo existem ainda 8 possibilidades,
pois o número não pode começar com 0 (senão seria um número de 2 algarismos). E para o
segundo algarismo também existem 8 possibilidades (o segundo algarismo pode ser 0).
                       8!       8!     8! 8! 8.7! 8.7!
   1. A8,1 . A8,1 =         .         = . =        .   = 8.8 = 64 números.
                    (8 − 1)! (8 − 1)! 7! 7! 7! 7!
Resposta : O número de divisíveis por 5 é 72 + 64 = 136 números.


6) Quantos são os números compreendidos entre 2000 e 3000 formados por algarismos
distintos escolhidos entre 1,2,3,4,5,6,7,8 e 9?
  R : O número deve ter quatro algarismos (pois está entre 2000 e 3000). Para o primeiro
algarismo existe apenas uma possibilidade (2), e para os outros três ainda existem 8 números
disponíveis, então :
               8!    8! 8.7.6.5!
  1. A8,3 =         = =            = 8.7.6 = 336 números.
            (8 − 3)! 5!       5!




                                                                                                 2
Permutação Simples: É um caso particular de arranjo simples. É o tipo de agrupamento ordenado onde
entram todos os elementos.


                                Pn = n!

7) Quantos números de 5 algarismos distintos podem ser formados por 1,2,3,5 e 8?
P5 = 5! = 5.4.3.2.1 = 120 números.


8) Quantos anagramas da palavra EDITORA :
a) COMEÇAM POR A.
  Para a primeira letra existe apenas uma possibilidade (A), e para as outras 6 letras
existem 6 possibilidades. Então o total é :
1.P6 = 1.6!= 6.5.4.3.2.1 = 720 anagramas.


b) COMEÇAM POR A e terminam com E.
  Para a primeira letra existe 1 possibilidade (A), e para última também só existe 1 (E),
e para as outras 5 letras existem 5 possibilidades. Então o total é :
1.1.P5 = 1.1.5!= 5.4.3.2.1 = 120 anagramas.


8) Calcule de quantas maneiras podem ser dipostas 4 damas e 4 cavalheiros, numa fila, de
forma que não fiquem juntos dois cavalheiros e duas damas.
   R :Existem duas maneiras de fazer isso :
     C - D - C - D - C - D - C - D ou D - C - D - C - D - C - D - C
Colocando um cavalheiro na primeira posição temos como número total de maneiras :
P4 .P4 = 4!.4!= 24.24 = 576 maneiras.
Colocando uma dama na primeira posição temos também :
P4 .P4 = 4!.4!= 24.24 = 576 maneiras.
Portanto o total é 576 + 576 = 1152 maneiras.

Combinação Simples: é o tipo de agrupamento em que um grupo difere do outro apenas pela natureza
dos elementos componentes.


                                          n!
                           Cn, p =
                                      p!(n − p )!




                                                                                                 3
9) Resolver a equação C m,3 − C m , 2 = 0.
    m!           m!
           −           =0
3!(m − 3)! 2!(m − 2)!
m.(m − 1).(m − 2).(m − 3)! m.(m − 1).(m − 2)!
                           −                     =0
        3!( m − 3)!              2!(m − 2)!
m.(m − 1).(m − 2) m.(m − 1)
                    −          =0
        3!              2!
m 3 − 2m 2 − m 2 + 2m m 2 − m
                      −          =0
           6                2
m 3 − 3m 2 + 2m − 3m 2 + 3m
                              = 0 ⇒ m 3 − 6m 2 + 5m = 0
              6
                           6 ± 16        m ' = 5
m 2 − 6m + 5 = 0 ⇒ m =               ⇒ 
                               2         m ' ' = 1
Resposta : m = 5.
obs : m = 1 não é a resposta porque não pode haver C1,3 .


10) Com 10 espécies de frutas, quantos tipos de salada, contendo 6 espécies diferentes
podem ser feitas?
             10!       10.9.8.7.6! 5040 5040
C10,6 =              =            =    =     = 210 tipos de saladas.
        6!.(10 − 6)!      6!.4!     4!   24

11) Numa reunião com 7 rapazes e 6 moças, quantas comissões podemos formar com 3
rapazes e 4 moças?
RAPAZES - C 7 ,3
MOÇAS - C 6, 4
O resultado é o produto C 7 ,3 .C 6, 4 .
    7!         6!       7.6.5.4! 6.5.4! 210 30
          .           =         .      =   . = 35.15 = 525 comissões.
3!(7 − 3)! 4!(6 − 4)!     3!.4! 4!.2!    3! 2




                                      TEORIA DOS CONJUNTOS
                                                                                         4
Símbolos


   : pertence                            : existe



   : não pertence                        : não existe



    : está contido                        : para todo (ou qualquer que seja)



    : não está contido                    : conjunto vazio



   : contém                            N: conjunto dos números naturais



   : não contém                        Z : conjunto dos números inteiros



/ : tal que                            Q: conjunto dos números racionais



     : implica que                     Q'= I: conjunto dos números irracionais



     : se, e somente se                R: conjunto dos números reais


         Veja também: Símbolos das operações - Conceitos sobre conjuntos




                          TABELA TRIGONOMÉTRICA




                                                                                 5
Ângulo      sen        cos         tg   Ângulo      sen        cos         tg
   1     0,017452   0,999848   0,017455 46        0,71934   0,694658    1,03553
   2     0,034899   0,999391   0,034921 47       0,731354   0,681998   1,072369
   3     0,052336    0,99863   0,052408 48       0,743145   0,669131   1,110613
   4     0,069756   0,997564   0,069927 49        0,75471   0,656059   1,150368
   5     0,087156   0,996195   0,087489 50       0,766044   0,642788   1,191754
   6     0,104528   0,994522   0,105104 51       0,777146    0,62932   1,234897
   7     0,121869   0,992546   0,122785 52       0,788011   0,615661   1,279942
   8     0,139173   0,990268   0,140541 53       0,798636   0,601815   1,327045
   9     0,156434   0,987688   0,158384 54       0,809017   0,587785   1,376382
  10     0,173648   0,984808   0,176327 55       0,819152   0,573576   1,428148
  11     0,190809   0,981627    0,19438   56     0,829038   0,559193   1,482561
  12     0,207912   0,978148   0,212557 57       0,838671   0,544639   1,539865
  13     0,224951    0,97437   0,230868 58       0,848048   0,529919   1,600335
  14     0,241922   0,970296   0,249328 59       0,857167   0,515038   1,664279
  15     0,258819   0,965926   0,267949 60       0,866025       0,5    1,732051
  16     0,275637   0,961262   0,286745 61        0,87462    0,48481   1,804048
  17     0,292372   0,956305   0,305731 62       0,882948   0,469472   1,880726
  18     0,309017   0,951057    0,32492   63     0,891007    0,45399   1,962611
  19     0,325568   0,945519   0,344328 64       0,898794   0,438371   2,050304
  20      0,34202   0,939693    0,36397   65     0,906308   0,422618   2,144507
  21     0,358368    0,93358   0,383864 66       0,913545   0,406737   2,246037
  22     0,374607   0,927184   0,404026 67       0,920505   0,390731   2,355852
  23     0,390731   0,920505   0,424475 68       0,927184   0,374607   2,475087
  24     0,406737   0,913545   0,445229 69        0,93358   0,358368   2,605089
  25     0,422618   0,906308   0,466308 70       0,939693    0,34202   2,747477
  26     0,438371   0,898794   0,487733 71       0,945519   0,325568   2,904211
  27      0,45399   0,891007   0,509525 72       0,951057   0,309017   3,077684
  28     0,469472   0,882948   0,531709 73       0,956305   0,292372   3,270853
  29      0,48481    0,87462   0,554309 74       0,961262   0,275637   3,487414
  30         0,5    0,866025    0,57735   75     0,965926   0,258819   3,732051
  31     0,515038   0,857167   0,600861 76       0,970296   0,241922   4,010781
  32     0,529919   0,848048   0,624869 77        0,97437   0,224951   4,331476
  33     0,544639   0,838671   0,649408 78       0,978148   0,207912    4,70463
  34     0,559193   0,829038   0,674509 79       0,981627   0,190809   5,144554
  35     0,573576   0,819152   0,700208 80       0,984808   0,173648   5,671282
  36     0,587785   0,809017   0,726543 81       0,987688   0,156434   6,313752
  37     0,601815   0,798636   0,753554 82       0,990268   0,139173    7,11537
  38     0,615661   0,788011   0,781286 83       0,992546   0,121869   8,144346
  39      0,62932   0,777146   0,809784 84       0,994522   0,104528   9,514364
  40     0,642788   0,766044     0,8391   85     0,996195   0,087156   11,43005
  41     0,656059    0,75471   0,869287 86       0,997564   0,069756   14,30067
  42     0,669131   0,743145   0,900404 87        0,99863   0,052336   19,08114
                                                                              6
Vetores
Reta Orientada - Eixo
  Uma reta r é orientada quando fixa nela um sentido de percurso, considerado positivo e indicado por uma
seta.




Segmento orientado
  Um segmento orientado é determinado por um par ordenado de pontos, o primeiro chamado origem do
segmento, o segundo chamado extremidade.




Segmento Nulo
  Um segmento nulo é aquele cuja extremidade coincide com a origem.


Segmentos Opostos
  Se AB é um segmento orientado, o segmento orientado BA é oposto de AB.


Medida de um Segmento
  Fixada uma unidade de comprimento, cada segmento orientado pode-se associar um número real, não
negativo, que é a medida do segmento em relação aquela unidade. A medida do segmento orientado é o seu
comprimento ou seu módulo. O comprimento do segmento AB é indicado por         .
  Assim, o comprimento do segmento AB representado na figura abaixo é de 5 unidades de comprimento:

           = 5 u.c.




  Observações

   a. Os segmentos nulos têm comprimento igual a zero

   b.      =      .

                                                Vetores
Direção e Sentido
  Dois segmentos orientados não nulos AB e CD têm a mesma direção se as retas suportes desses
segmentos são paralelas:




                                                                                                            7
ou coincidentes




  Observações

   a. Só se pode comparar os sentidos de dois segmentos orientados se eles têm mesma direção.
   b. Dois Segmentos orientados opostos têm sentidos contrários.


Segmentos Equipolentes
  Dois segmentos orientados AB e CD são equipolentes quando têm a mesma direção, o mesmo sentido e o
mesmo comprimento.
  Se os segmentos orientados AB e CD não pertencem à mesma reta. Na segunda figura abaixo, para que AB
seja equipolente a CD é necessário que AB//CD e AC/BD, isto é, ABCD deve ser um paralelogramo.




  Observações

   a. Dois segmentos nulos são sempre equipolentes.
   b. A equipolência dos segmentos AB e CD é representada por AB ~ CD.

                                                                                                       8
Propriedades da Equipolência

  I.     AB ~ AB (reflexiva).

 II.     Se AB ~ CD, CD ~ AB (simétrica).

III.     Se AB ~ CD e CD ~ EF, AB ~ EF (transitiva).

IV.      Dado o segmento orientado AB e um ponto C, existe um único ponto D tal que AB ~ CD.


 Vetor
 Vetor determinado por um segmento orientado AB é o conjunto de todos os segmentos orientados
equipolentes a AB.




  Se indicarmos com      este conjunto, simbolicamente poderemos escrever:

                                                   = {XY/XY ~ AB}
  onde XY é um segmento qualquer do conjunto.

 O vetor determinado por AB é indicado por          ou B - A ou   .

  um mesmo vetor       é determinado por uma infinidade de segmentos orientados, chamados representantes
desse vetor, e todos equipolentes entre si. Assim, um segmento determina um conjunto que é o vetor, e
qualquer um destes representantes determina o mesmo vetor. Usando um pouco mais nossa capacidade de
abstração, se considerarmos todos os infinitos segmentos orientados de origem comum, estaremos
caracterizando, através de representantes, a totalidade dos vetores do espaço. Ora, cada um destes
segmentos é um representante de um só vetor. Conseqüentemente, todos os vetores se acham representados
naquele conjunto que imaginamos.

  As características de um vetor     são as mesmas de qualquer um de seus representantes, isto é: o módulo,
a direção e o sentido do vetor são o módulo, direção e o sentido de qualquer um de seus representantes.

 O módulo de       se indica por | | .


 Vetores iguais

 Dois vetores       e     são iguais se, e somente se, AB ~ CD.


 Vetor Nulo
 Os segmentos nulos, por serem equipolentes entre si, determinam um único vetor, chamado vetor nulo ou
vetor zero, e que é indicado por      .


 Vetores Opostos

 Dado um vetor      =     , o vetor       é o oposto de    e se indica por   ou por     .
                                                                                                              9
Vetor Unitário

 Um vetor     é unitário se | | = 1.


Versor

 Versor de um vetor não nulo      é o vetor unitário de mesma direção e mesmo sentido de   .

 Por exemplo, tomemos um vetor         de módulo 3.




 Os vetores      e    da figura são vetores unitários, pois ambos têm módulo 1. No entanto, apenas   tem a
mesma direção e o mesmo sentido de        . Portanto, este é o versor de   .


Vetores Colineares

   Dois vetores e são colineares se tiverem a mesma direção. Em outras palavras:       e   são colineares se
tiverem representantes AB e CD pertencentes a uma mesma reta ou a retas paralelas.




Vetores Coplanares



                                                                                                          10
Se os vetores não nulos , e (não importa o número de vetores) possuem representantes AB, CD e EF
pertencentes a um mesmo plano π, diz-se que eles são coplanares.




  Dois vetores    e    quaisquer são são sempre coplanares, pois podemos sempre tomar um ponto no

espaço e, com origem nele, imaginar os dois representantes de          e    pertencendo a um plano p que passa por
este ponto.
 Três vetores poderão ou não ser coplanares.




                                              ,       e    são coplanares




                                          ,       e       não são coplanares



Soma de vetores
Se v=(a,b) e w=(c,d), definimos a soma de v e w, por:
v + w = (a+c,b+d)


Propriedades da soma de vetores



  I) Comutativa: Para todos os vetores u e v de R2:


                                                                                                               11
v+w=w+v
   II) Associativa: Para todos os vetores u, v e w de R2:
     u + (v + w) = (u + v) + w
   III) Elemento neutro: Existe um vetor O=(0,0) em R2 tal que para todo vetor u de R2, se tem:
     O+u=u
   IV) Elemento oposto: Para cada vetor v de R2, existe um vetor -v em R2 tal que:
     v + (-v) = O



Diferença de vetores
Se v=(a,b) e w=(c,d), definimos a diferença entre v e w, por:
v - w = (a-c,b-d)


Produto de um escalar por um vetor
Se v=(a,b) é um vetor e c é um número real, definimos a multiplicação de c por v, como:
c.v = (ca,cb)


Propriedades do produto de escalar por vetor
Quaisquer que sejam k e c escalares, v e w vetores:


                         •   1v=v
                         •   (k c) v = k (c v) = c (k v)
                         •   k v = c v implica k = c, se v for não nulo
                         •   k (v+w) = k v + k w

                         •   (k + c)v = k v + c v


Módulo de um vetor
O módulo ou comprimento do vetor v=(a,b) é um número real não negativo, definido por:




Vetor unitário
Vetor unitário é o que tem o módulo igual a 1.
Existem dois vetores unitários que formam a base canônica para o espaço R2, que são dados por:

i = (1,0)   j = (0,1)
Para construir um vetor unitário u que tenha a mesma direção e sentido que um outro vetor v, basta dividir o vetor v pelo
seu módulo, isto é:




Observação:
                                                                                                                            12
Para construir um vetor u paralelo a um vetor v, basta tomar u=cv onde c é um escalar não nulo. Nesse caso, u e v serão
paralelos.

Se c = 0 então u será o vetor nulo.
Se 0 < c < 1 então u terá comprimento menor do que v.
Se c > 1 então u terá comprimento maior do que v.
Se c < 0 então u terá sentido oposto ao de v.
                                          Próximo tópico:
   Produto escalar, Propriedades do produto escalar, Ângulos entre dois vetores, Vetores ortogonais




                                             PROBABILIDADE
   A história da teoria das probabilidades, teve início com os jogos de cartas, dados e de roleta. Esse é o
motivo da grande existência de exemplos de jogos de azar no estudo da probabilidade. A teoria da
probabilidade permite que se calcule a chance de ocorrência de um número em um experimento aleatório.
  Experimento Aleatório

                                                                                                                      13
É aquele experimento que quando repetido em iguais condições, podem fornecer resultados diferentes, ou
seja, são resultados explicados ao acaso. Quando se fala de tempo e possibilidades de ganho na loteria, a
abordagem envolve cálculo de experimento aleatório.
  Espaço Amostral
  É o conjunto de todos os resultados possíveis de um experimento aleatório. A letra que representa o espaço
amostral, é S.
  Exemplo:
   Lançando uma moeda e um dado, simultaneamente, sendo S o espaço amostral, constituído pelos 12
elementos:
  S = {K1, K2, K3, K4, K5, K6, R1, R2, R3, R4, R5, R6}

   1. Escreva explicitamente os seguintes eventos: A={caras e m número par aparece}, B={um número
      primo aparece}, C={coroas e um número ímpar aparecem}.
   2. Idem, o evento em que:

             a)       A ou B ocorrem;
             b)       B e C ocorrem;
             c)       Somente B ocorre.

   3. Quais dos eventos A,B e C são mutuamente exclusivos


Resolução:

   1. Para obter A, escolhemos os elementos de S constituídos de um K e um número par: A={K2, K4, K6};

        Para obter B, escolhemos os pontos de S constituídos de números primos: B={K2,K3,K5,R2,R3,R5}
        Para obter C, escolhemos os pontos de S constituídos de um R e um número ímpar: C={R1,R3,R5}.

   2. (a) A ou B = AUB = {K2,K4,K6,K3,K5,R2,R3,R5}

       (b) B e C = B ∩ C = {R3,R5}
       (c) Escolhemos os elementos de B que não estão em A ou C;

       B ∩ Ac ∩ Cc = {K3,K5,R2}


   3. A           e     C      são      mutuamente   exclusivos,   porque      A      ∩      C     =     ∅




Conceito de probabilidade
   Se em um fenômeno aleatório as possibilidades são igualmente prováveis, então a probabilidade de
   ocorrer um evento A é:



                                                                                                         14
Por, exemplo, no lançamento de um dado, um número par pode ocorrer de 3 maneiras diferentes dentre 6
    igualmente prováveis, portanto, P = 3/6= 1/2 = 50%
    Dizemos que um espaço amostral S (finito) é equiprovável quando seus eventos elementares têm
    probabilidades iguais de ocorrência.
    Num espaço amostral equiprovável S (finito), a probabilidade de ocorrência de um evento A é sempre:




Propriedades Importantes:
    1. Se A e A’ são eventos complementares, então:

    P( A ) + P( A' ) = 1
    2. A probabilidade de um evento é sempre um número entre ∅ (probabilidade de evento impossível) e 1
    (probabilidade do evento certo).




    Probabilidade Condicional
     Antes da realização de um experimento, é necessário que já tenha alguma informação sobre o evento que
    se deseja observar. Nesse caso, o espaço amostral se modifica e o evento tem a sua probabilidade de
    ocorrência alterada.
    Fórmula de Probabilidade Condicional
    P(E1 e E2 e E3 e ...e En-1 e En) é igual a P(E1).P(E2/E1).P(E3/E1 e E2)...P(En/E1 e E2 e ...En-1).
    Onde P(E2/E1) é a probabilidade de ocorrer E2, condicionada pelo fato de já ter ocorrido E1;
    P(E3/E1 e E2) é a probabilidade ocorrer E3, condicionada pelo fato de já terem ocorrido E1 e E2;
P(Pn/E1 e E2 e ...En-1) é a probabilidade de ocorrer En, condicionada ao fato de já ter ocorrido E1 e E2...En-1.


  Exemplo:
   Uma urna tem 30 bolas, sendo 10 vermelhas e 20 azuis. Se ocorrer um sorteio de 2 bolas, uma de cada vez
e sem reposição, qual será a probabilidade de a primeira ser vermelha e a segunda ser azul?
  Resolução:
  Seja o espaço amostral S=30 bolas, e considerarmos os seguintes eventos:
  A: vermelha na primeira retirada e P(A) = 10/30
  B: azul na segunda retirada e P(B) = 20/29
  Assim:
  P(A e B) = P(A).(B/A) = 10/30.20/29 = 20/87




  Eventos independentes
  Dizemos que E1 e E2 e ...En-1, En são eventos independentes quando a probabilidade de ocorrer um deles
não depende do fato de os outros terem ou não terem ocorrido.
                                                                                                                   15
Fórmula da probabilidade dos eventos independentes:
P(E1 e E2 e E3 e ...e En-1 e En) = P(E1).P(E2).p(E3)...P(En)


Exemplo:
 Uma urna tem 30 bolas, sendo 10 vermelhas e 20 azuis. Se sortearmos 2 bolas, 1 de cada vez e repondo a
 sorteada na urna, qual será a probabilidade de a primeira ser vermelha e a segunda ser azul?
 Resolução:
 Como os eventos são independentes, a probabilidade de sair vermelha na primeira retirada e azul na
 segunda retirada é igual ao produto das probabilidades de cada condição, ou seja, P(A e B) = P(A).P(B).
 Ora, a probabilidade de sair vermelha na primeira retirada é 10/30 e a de sair azul na segunda retirada
 20/30. Daí, usando a regra do produto, temos: 10/30.20/30=2/9.
 Observe que na segunda retirada forma consideradas todas as bolas, pois houve reposição. Assim, P(B/A)
 =P(B), porque o fato de sair bola vermelha na primeira retirada não influenciou a segunda retirada, já que
 ela foi reposta na urna.


 Probabilidade de ocorrer a união de eventos
 Fórmula da probabilidade de ocorrer a união de eventos:
 P(E1 ou E2) = P(E1) + P(E2) - P(E1 e E2)
 De fato, se existirem elementos comuns a E1 e E2, estes eventos estarão computados no cálculo de P(E1) e
 P(E2). Para que sejam considerados uma vez só, subtraímos P(E1 e E2).
 Fórmula de probabilidade de ocorrer a união de eventos mutuamente exclusivos:
 P(E1 ou E2 ou E3 ou ... ou En) = P(E1) + P(E2) + ... + P(En)


 Exemplo: Se dois dados, azul e branco, forem lançados, qual a probabilidade de sair 5 no azul e 3 no
 branco?
 Considerando os eventos:
 A: Tirar 5 no dado azul e P(A) = 1/6
 B: Tirar 3 no dado branco e P(B) = 1/6
 Sendo S o espaço amostral de todos os possíveis resultados, temos:
 n(S) = 6.6 = 36 possibilidades. Daí, temos:P(A ou B) = 1/6 + 1/6 – 1/36 = 11/36


 Exemplo: Se retirarmos aleatoriamente uma carta de baralho com 52 cartas, qual a probabilidade de ser
 um 8 ou um Rei?
 Sendo S o espaço amostral de todos os resultados possíveis, temos: n(S) = 52 cartas. Considere os
 eventos:
 A: sair 8 e P(A) = 4/52
 B: sair um rei e P(B) = 4/52
 Assim, P(A ou B) = 4/52 + 4/52 – 0 = 8/52 = 2/13. Note que P(A e B) = 0, pois uma carta não pode ser 8 e
 rei ao mesmo tempo. Quando isso ocorre dizemos que os eventos A e B são mutuamente exclusivos.




                                    DEFINIÇÃO DE LOGARITMO


                                                                                                           16
a x = b ⇔ x = log a b              sendo b>0 ,a>0 e a≠1

 Na igualdade x = log a b obtemos :
a= base do logaritmo
b= logaritmando ou antilogaritmo
x= logaritmo

Exemplos :
1) log 2 32 = 5 pois 2 5 = 32
2) log 4 16 = 2 pois 4 2 = 16
3) log 5 1 = 0 pois 5 0 = 1

                                             Consequências da definição

Sendo b>0 ,a>0 e a≠1 e m um número real qualquer, temos a seguir algumas consequências da
definição de logaritmo:

      log a 1 = 0              log a a = 1           log a a m = m          a log a b = b

                                log a b = log a c ⇔ b = c



                                       Propriedades operatórias dos logaritmos

1) Logaritmo do produto:               log a ( x. y ) = log a x + log a y    (a>0, a≠1, x>0 e y>0)

2) Logaritmo do quociente:                    x                    (a>0, a≠1, x>0 e y>0)
                                       log a   = log a x − log a y
                                              y
                                              

3) Logaritmo da potência:              log a x m = m. log a x        (a>0, a≠1, x>0 e m ∈ℜ)




                                             m
                           n
                               x   m
                                        =x   n


Caso particular: como , temos:

                       m
                               m
log a x = log a x =
      n    m           n
                                 . log a x
                               n


                                                      Cologaritmo

                                                                                                     17
Chamamos de cologaritmo de um número positivo b numa base a (a>0, a≠1) e indicamos
cologa b o logaritmo inverso desse número b na base a

                                         1
                     colog a b = log a         (a>0, a≠1 e b>0)
                                         b
             1
Como log a     = log a 1 − log a b = 0 − log a b = − log a b, podemos também escrever :
             b

                     colog a b = − log a b
                                             Mudança de base

        Em algumas situações podemos encontrar no cálculo vários logaritmos em bases diferentes.
Como as propriedades logarítmicas só valem para logaritmos numa mesma base, é necessário fazer,
antes, a conversão dos logaritmos de bases diferentes para uma única base conveniente. Essa
conversão chama-se mudança de base. Para fazer a mudança de uma base a para uma outra base b
usa-se:


                                                  log b x
                                      log a x =
                                                  log b a




                                   MATRIZES E DETERMINANTES




                                                                                             18
1) Dadas as matrizes :
      5 2         2 − 2         a b 
 A=        , B = 0 1  e X =  c d  tais que 2 A − X = B, calcule o determinante de X .
     − 1 1                          
 Primeiramente encontramos a matriz X :
   5     2   a b  2 − 2 
 2          −       =
   − 1   1   c d  0 1 
                         
  10     4  a b   2 − 2
 − 2        −       =
         2  c d  0 1 
                        
                                     10 − a = 2 → a = 8
                                     4 − b = −2 → b = 6
  10 − a 4 − b  2 − 2                                                8 6
  − 2 − c 2 − d  = 0 1     ⇒                                ⇒    X =      
                                 − 2 − c = 0 → c = −2               − 2 1 
                                     2 − d = 1 → d = 1
                                     
           8 6
 det X =        = 8.1 − 6.(−2) = 8 + 12 = 20
           −2 1




                                 2 1     3
2) Encontre a solução da equação 4 − 1 n − 1 = 12.
                                 n 0     n
Para achar o determinante de uma matriz 3x3 podemos utilizar a regra de Sarrus, que consiste em
copiar as duas primeiras colunas à direita da matriz, e subtrair a soma dos produtos da primeira
diagonal, pela soma dos produtos da segunda :
 2 1     3 2 1
 4 − 1 n − 1 4 − 1 = 12 ⇒ (−2n + n(n − 1) + 0) − ( −3n + 0 + 4n) = 12
 n 0     n n 0
(−2n + n 2 − n) − n = 12 ⇒ n 2 − 4n − 12 = 0
     4 ± 16-4.1.(-12 )      4 ± 64      4±8  n = 6
n=                     ⇒ n=        ⇒ n=     ⇒
           2                   2         2    n = −2

               1 0
                               5 − 3
3) Sendo A = − 2 3 e B = 
                                     calcule AB.
               0 4           1 2 
                     
Essa é uma questão de multiplicação de matrizes, onde estamos multiplicando uma matriz 3x2
por uma 2x2. O resultado será obtido pelo produto de cada linha da matriz A por cada coluna
da matriz B. O resultado será uma matriz 3x2.
      1.5 + 0.1    1.(−3) + 0.2          5 − 3
AB = (−2).5 + 3.1 (−2)(−3) + 3.2 ⇒ AB = − 7 12 
                                               
      0.5 + 4.1
                   0(−3) + 4.2         4
                                               8




                                                                                                   19
4 5 
4) Sendo A =     , determine a matriz inversa da matriz A.
             3 4
Sabemos que uma matriz multiplicada pela sua inversa resulta na matriz identidade, ou seja :
A. A −1 = I
                        4a + 5c = 1   4a + 5c = 1   a = 4
                        4b + 5d = 0               → 
4 5 a b  1 0                    3a + 4c = 0   c = −3
3 4. c d  = 0 1 ⇒ 3a + 4c = 0 ⇒ 4b + 5d = 0   b = −5
                                             → 
                                       
                        3b + 4d = 1
                                      3b + 4d = 1   d = 4
                                          4 − 5
Portanto, a matriz inversa de A é A −1 =       
                                         − 3 4 




                          O CONJUNTO DOS NÚMEROS COMPLEXOS




   De números complexos você deve saber :
   i 2 = −1
   Conjugado de um número complexo : z = a + bi ⇔ z = a − bi
                                                z1 z1 .z 2
   Divisão de dois números complexos :             =
                                                z 2 z 2 .z 2
   Módulo de um número complexo : z = a 2 + b 2
                                                              a             b
   Argumento de um número complexo : cos(θ ) =                  e sen(θ ) =
                                                              z             z
   Forma trigonométrica ou polar : z = z .(cos(θ ) + i. sen(θ ))
   Multiplicação na forma trigonométrica : z1 .z 2 = z1 . z 2 .(cos(θ 1 + θ 2 ) + i. sen(θ 1 + θ 2 ))
                                             z1   z1
   Divisão na forma trigonométrica :            =    .(cos(θ 1 − θ 2 ) + i. sen(θ 1 − θ 2 ))
                                             z2   z2
                                                          n
   Potenciação na forma trigonométrica : z n = z .(cos(nθ ) + i. sen(nθ ))




                                                                                                    20
Exercícios resolvidos

           2+i
1) Calcule       .
          5 − 3i
Multiplicam - se ambos os termos da fração pelo número complexo conjugado do
denominador :
 (2 + i ) (5 + 3i ) 10 + 6i + 5i + 3i 2 10 + 11i − 3 7 + 11i    7 11
         .          =                  =            =        =   + i
(5 − 3i ) (5 + 3i )     25 − 9i 2
                                         25 − ( −9)    34      34 34

                                       1− i    i
2) Coloque na forma a + bi a expressão      +     .
                                       1+ i i − 2
Em cada fração, multiplicamos seus termos pelo número complexo conjugado do
denominador :
(1 − i ) (1 − i )     i      ( −2 − i )     1 − 2i + i 2 − 2i − i 2   1 − 2i − 1 − 2i − (−1)
        .         +        .             =              +           =           +            =
(1 + i ) (1 − i ) (−2 + i ) (−2 − i )          1− i 2
                                                           4−i  2
                                                                       1 − (−1)   4 − (−1)
   − 2i 1 − 2i               1 − 2i       − 5i + 1 − 2i    1 − 7i     1 7
=         +         = −i+              =                 =          =   − i
    2         5                 5               5             5       5 5




                                                                                                 21
3) Calcule :
            92                4                                                        45   4
a) i   92
                 → 92         23       → i =1
                                          0
                                                                    b) i   45
                                                                                  → 44      11     → i1 = i
                    0                                                               1
                   310            4                                                 1081 4
c) i   310
                 → 308            77    → i = −1  2
                                                                    d) i   1081
                                                                                  → 1080 270 → i 1 = i
                    2                                                                 1
e) i 4 n = i 4 = i 2 .i 2 = (−1).(−1) = 1                            f) i 4 n +1 = i 4 n .i = 1.i = i
g) i 4 n + 2 = i 4 n .i 2 = 1.(−1) = −1                              h) i 4 n + 3 = i 4 n .i 3 = 1.(−i ) = −i


                                                             (4 − 3i )(12 − 5i )
4) Ache o módulo do número complexo                                                .
                                            2i
Primeiramente colocamos o número na forma a + bi :
(4 − 3i )(12 − 5i ) (− 2i )    (48 − 20i − 36i + 15i 2 ).(− 2i )   (33 − 56i ).(− 2i )
                   .         =                                   =                     =
     ( 2i )          (− 2i )                − 2i 2
                                                                        − 2(−1)
  − 33 2i − 56 2               33 2
=                  = − 28 2 −       i
         2                        2
Agora encontramos o módulo desse número complexo :
                                                              2
                        33 2           2178   8450                                                       4225
z = a + b = (−28 2 ) +  −
                 2
                       
                          2
                               = 1568 +      =
                                              2
                                                     =                                                          =
                          2            4       4                                                         2
    65            2       65 2                        65 2
=            .        =        →         z =
       2          2         2                           2

5) Obtenha o argumento dos números complexos a seguir :
a) z = 2 + 2 3i → z = 2 2 + (2 3 ) 2 = 4 + 12 = 16 = 4
                    a 2 1
cos(θ ) =            = =
                    z 4 2     π
                     θ = 60 =
                            0

         b 2 3    3           3
sen(θ ) = =    =
         z  4    2 




                                                                                                                    22
b) z = 4i → z = 0 2 + 4 2 = 16 = 4
         a 0    
cos(θ ) = = =0 
         z 4              π
                 θ = 90 =
                        0

         b 4               2
sen(θ ) = = = 1 
         z 4    
                

6) Passe o número complexo z = 8i para a forma trigonométrica.
z = 0 2 + 8 2 = 64 = 8
         a 0        
cos(θ ) =  = =0 
         z 8              π
                     θ =
         b 8                2
sen(θ ) = = = 1 
         z 8        
                    
Passando para a forma trigonométrica :
z = z .(cos(θ ) + i. sen(θ ))
       π              π 
z = 8. cos  + i. sen   
                            
       2              2 


                                                 π             π 
7) Dados z1 = 5(cos(π ) + i. sen(π )) e z 2 = 3. cos  + i. sen  , obtenha z1 .z 2 .
                                                                     
                                                 3             3 
z1 = (5 cos(π )) 2 + (5 sen(π )) 2 = (−5) 2 + 0 2 = 25 = 5
                      2                2
             π           π          9 27   36
z 2 =  3 cos   +  3 sen    =
                                         +   =    = 9 =3
             3           3          4 4     4
                                                              a    3/ 2 1         
            a    −5                            cos(θ 2 ) =      =     =          
cos(θ 1 ) =    =    = −1                                     z2    3    2
            z1    5                                                              
                                                                                        π
                         θ1 = π                                           3 3     θ2 =
            b    0                                                                       3
sen(θ 1 ) =    = =0                                                  b           
                                                                            2 = 3 
            z1 5        
                                                         sen(θ 2 ) =    =
                                                                      z2    3   2 
                                                                                  
z1 .z 2 = z1 . z 2 .(cos(θ 1 + θ 2 ) + i. sen(θ 1 + θ 2 ))
                      π              π 
z1 .z 2 = 5.3. cos π +  + i. sen  π +  
              
                      3              3 
                                            
               4π            4π   
z1 .z 2 = 15. cos
                    + i. sen       
                                       
               3             3    




                                                                                             23
POLINÔMIOS

   • Definição

    Uma função polinomial ou simplesmente polinômio, é toda função definida pela
relação P(x)=anxn + an-1.xn-1 + an-2.xn-2 + ... + a2x2 + a1x + a0.
    Onde:
    an, an-1, an-2, ..., a2, a1, a0 são números reais chamados coeficientes.
    n ∈ IN
    x ∈ C (nos complexos) é a variável.

   GRAU DE UM POLINÔMIO:

   Grau de um polinômio é o expoente máximo que ele possui. Se o coeficiente an≠0,
então o expoente máximo n é dito grau do polinômio e indicamos gr(P)=n. Exemplos:
   a) P(x)=5 ou P(x)=5.x0 é um polinômio constante, ou seja, gr(P)=0.
   b) P(x)=3x+5 é um polinômio do 1º grau, isto é, gr(P)=1.
   c) P(x)=4x5+7x4 é um polinômio do 5º grau, ou seja, gr(P)=5.

   Obs: Se P(x)=0, não se define o grau do polinômio.


   • Valor numérico

   O valor numérico de um polinômio P(x) para x=a, é o número que se obtém
substituindo x por a e efetuando todas as operações indicadas pela relação que define o
polinômio. Exemplo:
   Se P(x)=x3+2x2+x-4, o valor numérico de P(x), para x=2, é:
   P(x)= x3+2x2+x-4
   P(2)= 23+2.22+2-4
   P(2)= 14

   Observação: Se P(a)=0, o número a chamado raiz ou zero de P(x).
   Por exemplo, no polinômio P(x)=x2-3x+2 temos P(1)=0; logo, 1 é raiz ou zero desse
polinômio.




                                                                                    24
Alguns exercícios resolvidos:

1º) Sabendo-se que –3 é raiz de P(x)=x3+4x2-ax+1, calcular o valor de a.
Resolução: Se –3 é raiz de P(x), então P(-3)=0.
P(-3)=0 => (-3)3+4(-3)2-a.(-3)+1 = 0
3a = -10 => a=-10/3
Resposta: a=-10/3

2º) Calcular m ∈ IR para que o polinômio
P(x)=(m2-1)x3+(m+1)x2-x+4 seja:
a) do 3ºgrau         b) do 2º grau               c) do 1º grau

Resposta:
a) para o polinômio ser do 3º grau, os coeficientes de x2 e x3 devem ser diferentes de
   zero. Então:
   m2-1≠0 => m2≠1 => m≠1
   m+1≠0 => m≠-1
   Portanto, o polinômio é do 3º grau se m≠1 e m≠-1.

b) para o polinômio ser do 2º grau, o coeficiente de x3 deve ser igual a zero e o
   coeficiente de x2 diferente de zero. Então:
   m2-1=0 => m2=1 => m=±1
   m+1≠0 => m≠-1
   Portanto, o polinômio é do 2º grau se m=1.

c) para o polinômio ser do 1º grau, os coeficientes de x2 e x3 devem ser iguais a zero.
   Então:
   m2-1=0 => m2=1 => m=±1
   m+1=0 => m=-1
   Portanto, o polinômio é do 1º grau se m=-1.




                                                                                    25
3º) Num polinômio P(x), do 3º grau, o coeficiente de x3 é 1. Se P(1)=P(2)=0 e P(3)=30,
calcule o valor de P(-1).
Resolução:
Temos o polinômio: P(x)=x3+ax2+bx+c.
Precisamos encontrar os valores de a,b e c (coeficientes).
Vamos utilizar os dados fornecidos pelo enunciado do problema:


P(1)=0 => (1)3+a.(1)2+b(1)+c = 0 => 1+a+b+c=0 => a+b+c=-1
P(2)=0 => (2)3+a.(2)2+b(2)+c = 0 => 8+4a+2b+c=0 => 4a+2b+c=-8
P(3)=30 => (3)3+a.(3)2+b(3)+c = 30 => 27+9a+3b+c=30 => 9a+3b+c=3

Temos um sistema de três variáveis:
 a + b + c = -1
 
 4a + 2b + c = -8
 9a + 3b + c = 3
 

Resolvendo esse sistema encontramos as soluções:
a=9, b=-34, c=24
Portanto o polinômio em questão é P(x)= x3+9x2-34x+24.
O problema pede P(-1):
P(-1)= (-1)3+9(-1)2-34(-1)+24 => P(-1)=-1+9+34+24
P(-1)= 66
Resposta: P(-1)= 66




                                                                                   26
• Polinômios iguais

    Dizemos que dois polinômios A(x) e B(x) são iguais ou idênticos (e indicamos
A(x)≡B(x)) quando assumem valores numéricos iguais para qualquer valor comum
atribuído à variável x. A condição para que dois polinômios sejam iguais ou idênticos é
que os coeficientes dos termos correspondentes sejam iguais.
    Exemplo:
    Calcular a,b e c, sabendo-se que x2-2x+1 ≡ a(x2+x+1)+(bx+c)(x+1).
    Resolução: Eliminando os parênteses e somando os termos semelhantes do segundo
membro temos:
    x2-2x+1 ≡ ax2+ax+a+bx2+bx+cx+c
    1x2-2x+1 ≡ (a+b)x2+(a+b+c)x+(a+c)
    Agora igualamos os coeficientes correspondentes:
        a + b = 1
        
        a + b + c = −2
        a + c = 1
        


   Substituindo a 1ª equação na 2ª:
   1+c = -2 => c=-3.
   Colocando esse valor de c na 3ª equação, temos:
   a-3=1 => a=4.
   Colocando esse valor de a na 1ª equação, temos:
   4+b=1 => b=-3.
   Resposta: a=4, b=-3 e c=-3.

   Obs: um polinômio é dito identicamente nulo se tem todos os seus coeficientes
nulos.




                                                                                    27
• Divisão de polinômios

    Sejam dois polinômios P(x) e D(x), com D(x) não nulo.
    Efetuar a divisão de P por D é determinar dois polinômios Q(x) e R(x), que
satisfaçam as duas condições abaixo:
    1ª) Q(x).D(x) + R(x) = P(x)
    2ª) gr(R) < gr(D) ou R(x)=0
                                P( x)    D( x )
                                R( x)    Q( x)

   Nessa divisão:
   P(x) é o dividendo.
   D(x) é o divisor.
   Q(x) é o quociente.
   R(x) é o resto da divisão.

   Obs: Quando temos R(x)=0 dizemos que a divisão é exata, ou seja, P(x) é divisível
por D(x) ou D(x) é divisor de P(x).

                       Se D(x) é divisor de P(x) ⇔
                                  R(x)=0
Exemplo:
Determinar o quociente de P(x)=x4+x3-7x2+9x-1 por D(x)=x2+3x-2.
Resolução: Aplicando o método da chave, temos:

            x 4 + x3 − 7 x 2 + 9 x − 1            x 2 + 3x − 2
          − x 4 − 3x3 + 2 x 2                    x 2 − 2 x + 1 → Q( x)
              − 2 x3 − 5x2 + 9 x − 1
              + 2 x3 + 6 x2 − 4 x
                         x2 + 5x − 1
                      − x 2 − 3x + 2
                              2 x + 1 → R ( x)



   Verificamos que:

     x 4  -  1 ≡ (x 2 + 3x - 2) (x 2 - 2x + 1) + (2x + 1)
      + x 7x + 9x
           3   2
                   -
                                            
              P(x)                        D(x)              Q(x)         R(x)




                                                                                 28
• Divisão de um polinômio por um binômio da forma ax+b

  Vamos calcular o resto da divisão de P(x)=4x2-2x+3 por D(x)=2x-1.
  Utilizando o método da chave temos:

                    4 x2 − 2 x + 3     2x − 1
                  − 4 x2 + 2 x       2x
                                 3

  Logo: R(x)=3
  A raiz do divisor é 2x-1=0 => x=1/2.
  Agora calculamos P(x) para x=1/2.
  P(1/2) = 4(1/4) – 2(1/2) + 3
  P(1/2) = 3

  Observe que R(x) = 3 = P(1/2)
  Portanto, mostramos que o resto da divisão de P(x) por D(x) é igual ao valor
numérico de P(x) para x=1/2, isto é, a raiz do divisor.



  • Teorema do resto

O resto da divisão de um polinômio P(x) pelo binômio ax+b é igual a P(-b/a).

  Note que –b/a é a raiz do divisor.

  Exemplo: Calcule o resto da divisão de x2+5x-1 por x+1.
  Resolução: Achamos a raiz do divisor:
  x+1=0 => x=-1
  Pelo teorema do resto sabemos que o resto é igual a P(-1):
  P(-1)=(-1)2+5.(-1)-1 => P(-1) = -5 = R(x)
  Resposta: R(x) = -5.


  • Teorema de D’Alembert

      Um polinômio P(x) é divisível pelo binômio ax+b se P(-b/a)=0


   Exemplo: Determinar o valor de p, para que o polinômio P(x)=2x3+5x2-px+2 seja
divisível por x-2.
   Resolução: Se P(x) é divisível por x-2, então P(2)=0.
       P(2)=0 => 2.8+5.4-2p+2=0 => 16+20-2p+2=0 => p=19
   Resposta: p=19.
                                                                               29
• Divisão de um polinômio pelo produto (x-a)(x-b)

   Vamos resolver o seguinte problema: calcular o resto da divisão do polinômio P(x)
pelo produto (x-a)(x-b), sabendo-se que os restos da divisão de P(x) por (x-a) e por (x-b)
são, respectivamente, r1 e r2.
   Temos:
   a é a raiz do divisor x-a, portanto P(a)=r1     (eq. 1)
   b é a raiz do divisor x-b, portanto P(b)=r2     (eq. 2)
   E para o divisor (x-a)(x-b) temos P(x)=(x-a)(x-b) Q(x) + R(x)        (eq. 3)

   O resto da divisão de P(x) por (x-a)(x-b) é no máximo do 1º grau, pois o divisor é do
2º grau; logo:
   R(x)=cx+d

   Da eq.3 vem:
   P(x)=(x-a)(x-b) Q(x) + cx + d
   Fazendo:
   x=a => P(a) = c(a)+d         (eq. 4)
   x=b => P(b) = c(b)+d         (eq. 5)

   Das equações 1, 2, 4 e 5 temos:

           ca + d = r1
           
           cb + d = r2

   Resolvendo o sistema obtemos:

       r1 − r2         ar − ar1
    c=           e d= 2         , com a ≠ b
        a−b              a−b
                    r −r     ar − ar1
    Logo : R ( x) = 1 2 x + 2         , com a ≠ b
                     a−b       a−b


   Observações:
   1ª) Se P(x) for divisível por (x-a) e por (x-b), temos:
   P(a)= r1 =0
   P(b)= r2 =0
   Portanto, P(x) é divisível pelo produto (x-a)(x-b), pois:

                 r1 − r2   ar − ar1
       R( x) =           x+ 2       = 0+0 = 0
                  a −b       a−b


                                                                                       30
2ª) Generalizando, temos:
       Se P(x) é divisível por n fatores distintos (x-a1), (x-a2),..., (x-an) então P(x) é
divisível pelo produto (x-a1)(x-a2)...(x-an).

   Exemplo:
   Um polinômio P(x) dividido por x dá resto 6 e dividido por (x-1) dá resto 8. Qual o
resto da divisão de P(x) por x(x-1)?
   Resolução:
   0 é a raiz do divisor x, portanto P(0)=6   (eq. 1)

   1 é a raiz do divisor x-1, portanto P(1)=8   (eq. 2)
   E para o divisor x(x-1) temos P(x)=x(x-1) Q(x) + R(x)         (eq. 3)

   O resto da divisão de P(x) por x(x-1) é no máximo do 1º grau, pois o divisor é do 2º
grau; logo:
   R(x)=ax+b

   Da eq.3 vem:
   P(x)=x(x-1) Q(x) + ax + b
   Fazendo:
   x=0 => P(0) = a(0)+b => P(0) = b            (eq. 4)
   x=1 => P(1) = a(1)+b => P(1) = a+b          (eq. 5)

   Das equações 1, 2, 4 e 5 temos:

           b = 6
           
           a + b = 8

   Logo, b=6 e a=2.
   Agora achamos o resto: R(x) = ax+b = 2x+6
   Resposta: R(x) = 2x+6.




                                                                                       31
• O dispositivo de Briot-Ruffini

   Serve para efetuar a divisão de um polinômio P(x) por um binômio da forma (ax+b).
   Exemplo: Determinar o quociente e o resto da divisão do polinômio
P(x)=3x3-5x2+x-2 por (x-2).
   Resolução:
           RAIZ DO DIVISOR
             
                           ES DE P(x) 
                                   COEFICIENT
                                                 
              2              3    −5          1     −2
                             ↓         3.(2) − 5     1.( 2) + 1   3.( 2) − 2

                             1 
                             3        3
                                                                  4
                                                                  
                              COEFICIENTES DO QUOCIENTE Q(x)      RESTO




   Observe que o grau de Q(x) é uma unidade inferior ao de P(x), pois o divisor é de
grau 1.
   Resposta: Q(x)=3x2+x+3 e R(x)=4.

   Para a resolução desse problema seguimos os seguintes passos:
   1º) Colocamos a raiz do divisor e os coeficientes do dividendo ordenadamente na
parte de cima da “cerquinha”.
   2º) O primeiro coeficiente do dividendo é repetido abaixo.
   3º) Multiplicamos a raiz do divisor por esse coeficiente repetido abaixo e somamos o
produto com o 2º coeficiente do dividendo, colocando o resultado abaixo deste.
   4º) Multiplicamos a raiz do divisor pelo número colocado abaixo do 2º coeficiente e
somamos o produto com o 3º coeficiente, colocando o resultado abaixo deste, e assim
sucessivamente.
   5º) Separamos o último número formado, que é igual ao resto da divisão, e os
números que ficam à esquerda deste serão os coeficientes do quociente.




                                                                                    32
• Decomposição de um polinômio em fatores

    Vamos analisar dois casos:
    1º caso: O polinômio é do 2º grau.
       De uma forma geral, o polinômio de 2º grau P(x)=ax2+bx+c que admite as raízes
r1 e r2 pode ser decomposto em fatores do 1º grau, da seguinte forma:

                  ax2+bx+c = a(x-r1)(x-r2)

      Exemplos:
      1) Fatorar o polinômio P(x)=x2-4.
         Resolução: Fazendo x2-4=0, obtemos as raízes r1=2 e r2=-2.
         Logo: x2-4 = (x-2)(x+2).

      2) Fatorar o polinômio P(x)=x2-7x+10.
         Resolução: Fazendo x2-7x+10=0, obtemos as raízes r1=5 e r2=2.
         Logo: x2-7x+10 = (x-5)(x-2).

    2º caso: O polinômio é de grau maior ou igual a 3.
       Conhecendo uma das raízes de um polinômio de 3º grau, podemos decompô-lo
num produto de um polinômio do 1º grau por um polinômio do 2º grau e, se este tiver
raízes, podemos em seguida decompô-lo também.

   Exemplo: Decompor em fatores do 1º grau o polinômio 2x3-x2-x.
   Resolução:
     2x3-x2-x = x.(2x2-x-1)  colocando x em evidência
     Fazendo x.(2x2-x-1) = 0 obtemos: x=0 ou 2x2-x-1=0.
     Uma das raízes já encontramos (x=0).
     As outras duas saem da equação: 2x2-x-1=0 => r1=1 e r2=-1/2.
     Portanto, o polinômio 2x3-x2-x, na forma fatorada é:
     2.x.(x-1).(x+(1/2)).

Generalizando, se o polinômio P(x)=anxn+an-1xn-1+...+a1x+a0 admite n raízes r1, r2,..., rn,
podemos decompô-lo em fatores da seguinte forma:

          anxn+an-1xn-1+...+a1x+a0 = an(x-r1)(x-r2)...(x-rn)



Observações:
     1) Se duas, três ou mais raiz forem iguais, dizemos que são raízes duplas, triplas,
        etc.
     2) Uma raiz r1 do polinômio P(x) é dita raiz dupla ou de multiplicidade 2 se P(x)
        é divisível por (x-r1)2 e não por (x-r1)3.


                                                                                        33
PRODUTOS NOTÁVEIS
      É muito comum nas expressões algébrica o aparecimento de certos produtos. Para
simplificar o trabalho nos cálculos será muito útil a aplicação dos produtos notáveis.
Veja a tabela abaixo:

              Produtos notáveis                                 Exemplos
       2      2           2                               2     2
(a+b) = a +2ab+b                                    (x+3) = x +6x+9
(a-b)2 = a2-2ab+b2                                  (x-3)2 = x2-6x+9
(a+b)(a-b) = a2-b2                                  (x+3)(x-3) = x2-9
(x+a)(x+b) = x2+(a+b)x+ab                           (x+2)(x+3) = x2+5x+6
(a+b)3 = a3+3a2b+3ab2+b3                            (x+2)3 = x3+6x2+12x+8
(a-b)3 = a3-3a2b+3ab2-b3                            (x-2)3 = x3-6x2+12x-8
(a+b)(a2-ab+b2) = a3+b3                             (x+2)(x2-2x+4) = x3+8
(a-b)(a2+ab+b2) = a3-b3                             (x-2)(x2+2x+4) = x3-8


ALGUNS EXERCÍCIOS RESOLVIDOS:

1) Desenvolva:
   a) (3x+y)2
   (3x+y)2 = (3x)2+2.3x.y+y2 = 9x2+6xy+y2

   b) ((1/2)+x2)2
   ((1/2)+x2)2 = (1/2)2+2.(1/2).x2+(x2)2 = (1/4) +x2+x4

   c) ((2x/3)+4y3)2
   ((2x/3)+4y3)2 = (2x/3)2-2.(2x/3).4y3+(4y3)2= (4/9)x2-(16/3)xy3+16y6

   d) (2x+3y)3
   (2x+3y)3 = (2x)3+3.(2x)2.3y+3.2x.(3y)2+(3y)3 = 8x3+36x2y+54xy2+27y3

   e) (x4+(1/x2))3
   (x4+(1/x2))3 = (x4)3+3.(x4)2.(1/x2)+3.x4.(1/x2)2+(1/x2)3 = x12+3x6+3+(1/x6)

   f) ((2x/3)+(4y/5)).((2x/3)-(4y/5))
   ((2x/3)+(4y/5)).((2x/3)-(4y/5)) = (2x/3)2-(4y/5)2 = (4/9)x2-(16/25)y2

2) Efetue as multiplicações:
   a) (x-2)(x-3)
   (x-2)(x-3) = x2+((-2)+(-3))x+(-2).(-3) = x2-5x+6

   b) (x+5)(x-4)
   (x+5)(x-4) = x2+(5+(-4))x+5.(-4) = x2+x-20
                                                                                   34
3) Simplifique as expressões:
   a) (x+y)2–x2-y2
   (x+y)2–x2-y2 = x2+2xy+y2–x2-y2 = 2xy

  b) (x+2)(x-7)+(x-5)(x+3)
  (x+2)(x-7)+(x-5)(x+3) = x2+(2+(-7))x+2.(-7) + x2+(-5+3)x+3.(-5) =
  x2-5x-14+ x2-2x-15 = 2x2-7x-29

  c) (2x-y)2-4x(x-y)
  (2x-y)2-4x(x-y) = (2x)2-2.2x.y+y2-4x2+4xy = 4x2-4xy+y2-4x2+4xy = y2




                                                                        35
Progressões Aritméticas
        Progressão aritmética é uma sequência numérica na qual, a partir do segundo, cada termo é
igual à soma de seu antecessor com uma constante, denominada razão.

      Fórmula do termo geral de uma P.A. : a n = a1 + (n − 1).r

                                                              (a1 + a n ).n
      Soma de termos de uma P.A. finita : S n =
                                                                   2
      Logo abaixo temos alguns exercícios de progressões aritméticas resolvidos.

1) Dada a P.A. (-19,-15,-11,...) calcule o seu enésimo termo.

Primeiramente encontramos a razão : r = a2 − a1 ⇒ r = −15 − (−19) ⇒ r = 4.
Logo, o termo geral é :
an = a1 + (n − 1).r ⇒ an = −19 + (n − 1).4 ⇒ an = −19 + 4n − 4 ⇒ an = 4n − 23


2) Interpole seis meios aritméticos entre –8 e 13.

No problema : a1 = −8, an = 13, n = 8 (pois 6 meios aritméticos serão interpolados
entre os dois extremos, que são - 8 e 13. Logo, existem 8 termos na P.A.).
Para interpolar os valores, devemos encontrar a razão :
an = a1 + (n − 1).r ⇒ 13 = −8 + (8 − 1).r ⇒ 13 = −8 + 7 r ⇒ 13 + 8 = 7 r ⇒
                   21
 7r = 21 ⇒ r =         ⇒ r = 3.
                   7
Encontrada a razão, basta interpolar os meios aritméticos :
- 8, - 5, - 2, 1, 4, 7, 10, 13




                                                                                              36
3) Escreva uma P.A. de três termos, sabendo que a soma desses termos vale 12 e que a
   soma de seus quadrados vale 80.

a1 + a 2 + a 3 = 12

 2
a1 + a 2 2 + a3 2 = 80

Sabemos que a 2 = a1 + r e que a 3 = a1 + 2r. Então substituimos no sistema acima :
a1 + (a1 + r ) + (a1 + 2r ) = 12   3a1 + 3r = 12
 2                               ⇒  2      2               2
                                                                                ⇒
a1 + (a1 + r ) + (a1 + 2r ) = 80   a1 + a1 + 2a1 r + r + a1 + 4a1 r + 4r = 80
                2              2                        2                 2




                                             12 − 3r
  3a1 + 3r = 12         →             a1 =             →     a1 = 4 − r
⇒                                               3
  3a 2 + 6a r + 5r 2 = 80
   1       1



Substituindo na segunda equação temos :
3(4 − r ) 2 + 6(4 − r )r + 5r 2 = 80
3(16 − 8r + r 2 ) + (24 − 6r )r + 5r 2 = 80
48 − 24r + 3r 2 + 24r − 6r 2 + 5r 2 = 80
48 + 2r 2 = 80 → 2r 2 = 80 − 48 → 2r 2 = 32 → r 2 = 16 → r = 16 → r = ±4
Agora encontramos o primeiro termo :
1) Para r = 4 :
a1 = 4 - r → a 1 = 4 - 4 → a 1 = 0
P.A : (0,4,8)

1) Para r = −4 :
a1 = 4 - r → a 1 = 4 - (-4) → a 1 = 8
P.A : (8,4,0)

Resposta : (0,4,8) ou (8,4,0).

4) Calcule quantos números inteiros existem entre 13 e 247 que não são múltiplos de 3.

  Entre 13 e 247 existem 233 números. Para calcular quantos números NÃO são múltiplos de 3,
  nós devemos calcular primeiramente quantos números SÃO múltiplos de 3, e logo após subtrair o número
  total de números (233) pelo número de múltiplos, o que dará como resultado o número de NÃO múltiplos.


  Para calcular o número de múltiplos de 3 :
  a1 = 15 (pois é o primeiro múltiplo de 3 depois do 13)
  r = 3,   a n = 246 (pois é o último múltiplo de 3 antes do 247). Basta achar o n, que é o número de múltiplos :
                                                                             234
  a n = a1 + (n − 1).r → 246 = 15 + (n - 1)3 → 231 = 3n - 3 → n =                  → n = 78
                                                                              3
  Dos 233 números, 78 são múltiplos de 3, logo 155 não são múltiplos de 3.




                                                                                                                    37
5) Encontre o valor de x para que a sequência (2x, x+1, 3x) seja uma progressão aritmética.


Para ser uma P.A. : a3 − a 2 = a 2 − a1
3 x − ( x + 1) = ( x + 1) − 2 x
2x − 1 = 1 − x
                                                  2
2x + x = 1 + 1     →       3x = 2      →     x=
                                                  3
6) Numa progressão aritmética em que a2+a7=a4+ak, o valor de k é:

(a1 + r ) + (a1 + 6r ) = (a1 + 3r ) + a k
2a1 + 7 r = a1 + 3r + a k
2a1 − a1 + 7 r − 3r = a k         → a k = a1 + 4r
Logo k = 5, pois a5 = a1 + 4r.


7) Se Sn é a soma dos n primeiros termos da progressão aritmética (-90,-86,-82,...) então o
menor valor de n para que se tenha Sn>0 é:

                                             r = 4
                                             
Pelo enunciado, obtemos os seguintes dados : a1 = −90
                                             a = 94 (pois a S deve ser maior que zero)
                                              n              n

Basta encontrar o número de termos :
a n = a1 + (n − 1).r
94 = −90 + (n − 1).4
94 + 90 = 4n − 4
                           188
184 + 4 = 4n → n =             → n = 47
                            4

8) A soma dos n primeiros números pares positivos é 132. Encontre o valor de n.

     r = 2 ; a1 = 2 ; S n = 132
     a n = a1 + (n − 1).r → a n = 2 + (n − 1).2 → a n = 2 + 2n − 2 → a n = 2n
     Substituindo na fórmula da soma temos :
          ( a + a n ).n          ( 2 + 2n) n
     Sn = 1             → 132 =                → n 2 + n − 132 = 0
               2                      2
         − 1 ± 1 + 4.1.132    − 1 ± 529        − 1 ± 23 n = −12
     n=                    =                 =         =          ⇒ n = 11
                  2                 2              2     n = 11




                                  PROGRESSÕES GEOMÉTRICAS
                                                                                          38
Podemos definir progressão geométrica, ou simplesmente P.G., como uma sucessão de
números reais obtida, com exceção do primeiro, multiplicando o número anterior por uma
quantidade fixa q, chamada razão.
   Podemos calcular a razão da progressão, caso ela não esteja suficientemente evidente,
dividindo entre si dois termos consecutivos. Por exemplo, na sucessão (1, 2, 4, 8,...), q = 2.
        Cálculos do termo geral
        Numa progressão geométrica de razão q, os termos são obtidos, por definição, a
partir do primeiro, da seguinte maneira:

   a1           a2            a3            ...          a20      ...      an           ...
   a1          a1xq         a1xq2          ...          a1xq19          a1xqn-1         ...
      Assim, podemos deduzir a seguinte expressão do termo geral, também chamado
enésimo termo, para qualquer progressão geométrica.

                                                 an = a1 x qn-1

 Portanto, se por exemplo, a1 = 2 e q = 1/2, então:

                                            an = 2 x (1/2)n-1

 Se quisermos calcular o valor do termo para n = 5, substituindo-o na fórmula, obtemos:

                             a5 = 2 x (1/2)5-1 = 2 x (1/2)4 = 1/8
    A semelhança entre as progressões aritméticas e as geométricas é aparentemente
grande. Porém, encontramos a primeira diferença substancial no momento de sua definição.
Enquanto as progressões aritméticas formam-se somando-se uma mesma quantidade de
forma repetida, nas progressões geométricas os termos são gerados pela multiplicação,
também repetida, por um mesmo número. As diferenças não param aí.
       Observe que, quando uma progressão aritmética tem a razão positiva, isto é, r > 0,
cada termo seu é maior que o anterior. Portanto, trata-se de uma progressão crescente. Ao
contrário, se tivermos uma progressão aritmética com razão negativa, r < 0, seu
comportamento será decrescente. Observe, também, a rapidez com que a progressão
cresce ou diminui. Isto é conseqüência direta do valor absoluto da razão, |r|. Assim, quanto
maior for r, em valor absoluto, maior será a velocidade de crescimento e vice-versa.
   Soma dos n primeiros termos de uma PG
  Seja a PG (a1, a2, a3, a4, ... , an , ...) .

Para o cálculo da soma dos n primeiros termos Sn,
Vamos considerar o que segue:
Sn = a1 + a2 + a3 + a4 + ... + an-1 + an
Multiplicando ambos os membros pela razão q vem:
Sn.q = a1 . q + a2 .q + .... + an-1 . q + an .q
Conforme a definição de PG, podemos reescrever a expressão como:
Sn . q = a2 + a3 + ... + an + an . q

                                                                                              39
Observe que a2 + a3 + ... + an é igual a Sn - a1 . Logo, substituindo, vem:
S n . q = S n - a1 + a n . q
Daí, simplificando convenientemente, chegaremos à seguinte fórmula da soma:




Se substituirmos an = a1 . qn-1 , obteremos uma nova apresentação para a fórmula da
soma, ou seja:




Exemplo:
Calcule a soma dos 10 primeiros termos da PG (1,2,4,8,...)
Temos:




Observe que neste caso a1 = 1.
   •   Soma dos termos de uma PG decrescente e ilimitada
Considere uma PG ILIMITADA ( infinitos termos) e decrescente. Nestas condições, podemos
considerar que no limite teremos an = 0. Substituindo na fórmula anterior, encontraremos:




Exemplo:
Resolva a equação: x + x/2 + x/4 + x/8 + x/16 + ... =100
O primeiro membro é uma PG de primeiro termo x e razão 1/2. Logo, substituindo na
fórmula, vem:




Dessa equação encontramos como resposta x = 50.

                                                                                      40
IDENTIDADES TRIGONOMÉTRICAS
             sen( x)                                                  π
1) tg ( x) =                          Relação válida para todo x ≠      + kπ
             cos( x)                                                  2
                cos( x)
2) cot g ( x) =                        Relação válida para todo x ≠ kπ
                sen( x)
                 1                                                    π
3) sec( x) =                           Relação válida para todo x ≠     + kπ
              cos( x)                                                 2
                     1
4) cos ec( x) =                        Relação válida para todo x ≠ kπ
                 sen( x)
5) sen 2 ( x) + cos 2 ( x) = 1


                              Fórmulas da Adição

    6) sen(a + b) = sen(a). cos(b) + sen(b). cos(a )
    7) sen(a − b) = sen(a). cos(b) − sen(b). cos(a )
    8) cos(a + b) = cos(a). cos(b) − sen(a). sen(b)
    9) cos(a − b) = cos(a ). cos(b) + sen(a). sen(b)
                                                      π
                                               p/ a ≠ + kπ
                                                        2
                         tg (a ) + tg (b)   
                                                      π
    10) tg ( a + b) =                        p/ b ≠ + kπ
                        1 − tg (a).tg (b)              2
                                            p/ (a + b) ≠ π + kπ
                                            
                                                         2
                                                      π
                                               p/ a ≠ + kπ
                                                        2
                         tg (a) − tg (b)    
                                                      π
    11) tg ( a − b) =                        p/ b ≠ + kπ
                        1 + tg (a).tg (b)              2
                                            p/ (a − b) ≠ π + kπ
                                            
                                                         2

    As fórmulas acima são verdadeiras para arcos positivos, cuja soma
    pertence ao primeiro quadrante.




                                                                               41
Fórmulas da Multiplicação

     12) sen(2 x) = 2. sen( x). cos( x)
     13) cos(2 x) = cos 2 ( x) − sen 2 ( x)
                      2.tg ( x)
     14) tg (2 x) =
                    1 − tg 2 ( x)

         Fórmulas da Transformação em Produto

                                x+ y     x− y
15) sen( x) + sen( y ) = 2. sen     . cos    
                                 2        2 

                            x− y     x+ y
16) sen(x) - sen(y) = 2. sen    . cos    
                             2        2 

                               x+ y     x− y
17) cos( x) + cos( y ) = 2. cos    . cos    
                                2        2 

                                x+ y     x− y
18) cos( x) − cos( y ) = −2. sen    . sen    
                                 2        2 




                                                    42
FUNÇÃO DE 1º GRAU
 Definição
 Chama-se função polinomial do 1º grau, ou função afim, a qualquer função f de IR em IR dada por uma lei
da forma f(x) = ax + b, onde a e b são números reais dados e a 0.
Na função f(x) = ax + b, o número a é chamado de coeficiente de x e o número b é chamado termo constante.
Veja alguns exemplos de funções polinomiais do 1º grau:
f(x) = 5x - 3, onde a = 5 e b = - 3
f(x) = -2x - 7, onde a = -2 e b = - 7
f(x) = 11x, onde a = 11 e b = 0


Gráfico

  O gráfico de uma função polinomial do 1º grau, y = ax + b, com a       0, é uma reta oblíqua aos eixos Ox e Oy.
  Exemplo:
  Vamos construir o gráfico da função y = 3x - 1:
  Como o gráfico é uma reta, basta obter dois de seus pontos e ligá-los com o auxílio de uma régua:
  a)   Para x = 0, temos y = 3 · 0 - 1 = -1; portanto, um ponto é (0, -1).


  b)   Para y = 0, temos 0 = 3x - 1; portanto,         e outro ponto é          .



  Marcamos os pontos (0, -1) e           no plano cartesiano e ligamos os dois com uma reta.




       x                 y
       0                 -1

                         0




   Já vimos que o gráfico da função afim y = ax + b é uma reta.
   O coeficiente de x, a, é chamado coeficiente angular da reta e, como veremos adiante, a está ligado à
inclinação da reta em relação ao eixo Ox.
  O termo constante, b, é chamado coeficiente linear da reta. Para x = 0, temos y = a · 0 + b = b. Assim, o
coeficiente linear é a ordenada do ponto em que a reta corta o eixo Oy.




                                                                                                              43
Zero e Equação do 1º Grau
  Chama-se zero ou raiz da função polinomial do 1º grau f(x) = ax + b, a 0, o número real x
tal que f(x) = 0.
 Temos:


 f(x) = 0       ax + b = 0
 Vejamos alguns exemplos:
   1. Obtenção do zero da função f(x) = 2x - 5:

                             f(x) = 0       2x - 5 = 0
   2. Cálculo da raiz da função g(x) = 3x + 6:
                             g(x) = 0        3x + 6 = 0      x = -2

   3. Cálculo da abscissa do ponto em que o gráfico de h(x) = -2x + 10 corta o eixo das
      abicissas:
      O ponto em que o gráfico corta o eixo dos x é aquele em que h(x) = 0; então:
        h(x) = 0       -2x + 10 = 0        x=5


Crescimento e decrescimento
  Consideremos a função do 1º grau y = 3x - 1. Vamos atribuir valores cada vez maiores a x
e observar o que ocorre com y:




                                                                                          44
x              -3            -2            -1             0              1              2         3
        y             -10            -7            -4             -1             2              5         8




   Notemos que, quando aumentos o valor de x, os correspondentes valores de y também
aumentam. Dizemos, então que a
  função y = 3x - 1 é crescente.




Observamos novamente seu gráfico:




Regra geral:
a função do 1º grau f(x) = ax + b é crescente quando o coeficiente de x é positivo (a > 0);
a função do 1º grau f(x) = ax + b é decrescente quando o coeficiente de x é negativo (a < 0);
Justificativa:

    •       para a > 0: se x1 < x2, então ax1 < ax2. Daí, ax1 + b < ax2 + b, de onde vem f(x1) < f(x2).
    •       para a < 0: se x1 < x2, então ax1 > ax2. Daí, ax1 + b > ax2 + b, de onde vem f(x1) > f(x2).

Sinal
  Estudar o sinal de uma qualquer y = f(x) é determinar os valor de x para os quais y é positivo, os valores de x
para os quais y é zero e os valores de x para os quais y é negativo.
  Consideremos uma função afim y = f(x) = ax + b vamos estudar seu sinal. Já vimos que essa função se


anula pra raiz              . Há dois casos possíveis:
                                                                                                               45
1º) a > 0 (a função é crescente)


     y>0          ax + b > 0          x>


     y<0         ax + b < 0           x<
   Conclusão: y é positivo para valores de x maiores que a raiz; y é negativo para valores de x menores que a
raiz




2º) a < 0 (a função é decrescente)


      y>0      ax + b > 0             x<


     y<0       ax + b < 0        x>


Conclusão: y é positivo para valores de x menores que a raiz; y é negativo para valores de x maiores que a
raiz.




                                                                                                             46
FUNÇÃO QUADRÁTICA
 Definição
  Chama-se função quadrática, ou função polinomial do 2º grau, qualquer função f de IR em
IR dada por uma lei da forma f(x) = ax2 + bx + c, onde a, b e c são números reais e a 0.
  Vejamos alguns exemplos de função quadráticas:

   1.   f(x) = 3x2 - 4x + 1, onde a = 3, b = - 4 e c = 1
   2.   f(x) = x2 -1, onde a = 1, b = 0 e c = -1
   3.   f(x) = 2x2 + 3x + 5, onde a = 2, b = 3 e c = 5
   4.   f(x) = - x2 + 8x, onde a = 1, b = 8 e c = 0
   5.   f(x) = -4x2, onde a = - 4, b = 0 e c = 0



Gráfico
  O gráfico de uma função polinomial do 2º grau, y = ax2 + bx + c, com a   0, é uma curva
chamada parábola.
Exemplo:
  Vamos construir o gráfico da função y = x2 + x:
  Primeiro atribuímos a x alguns valores, depois calculamos o valor correspondente de y e,
em seguida, ligamos os pontos assim obtidos.




                                                                                            47
x               y
             -3              6
             -2              2
             -1              0



             0               0
             1               2
             2               6




  Observação:
 Ao construir o gráfico de uma função quadrática y = ax2 + bx + c, notaremos sempre que:
   •   se a > 0, a parábola tem a concavidade voltada para cima;
   •   se a < 0, a parábola tem a concavidade voltada para baixo;




Zero e Equação do 2º Grau
  Chama-se zeros ou raízes da função polinomial do 2º grau f(x) = ax2 + bx + c , a   0, os
números reais x tais que f(x) = 0.
  Então as raízes da função f(x) = ax2 + bx + c são as soluções da equação do 2º grau ax2 +
bx + c = 0, as quais são dadas pela chamada fórmula de Bhaskara:




  Temos:




Observação
 A quantidade de raízes reais de uma função quadrática depende do valor obtido para o
radicando               , chamado discriminante, a saber:
                                                                                             48
•   quando é positivo, há duas raízes reais e distintas;
   •   quando é zero, há só uma raiz real;
   •   quando é negativo, não há raiz real.




                              Função Quadrática

 Coordenadas do vértice da parábola
  Quando a > 0, a parábola tem concavidade voltada para cima e um ponto de mínimo V;
quando a < 0, a parábola tem concavidade voltada para baixo e um ponto de máximo V.


Em qualquer caso, as coordenadas de V são             . Veja os gráficos:




                                                                                       49
Imagem
   O conjunto-imagem Im da função y = ax2 + bx + c, a   0, é o conjunto dos valores que y
pode assumir. Há duas possibilidades:
1ª - quando a > 0,




                                                                                        50
a>0



2ª quando a < 0,




                   a<0




Construção da Parábola
 É possível construir o gráfico de uma função do 2º grau sem montar a tabela de pares
(x, y), mas seguindo apenas o roteiro de observação seguinte:
   1. O valor do coeficiente a define a concavidade da parábola;

                                                                                        51
2. Os zeros definem os pontos em que a parábola intercepta o eixo dos x;


       3. O vértice V              indica o ponto de mínimo (se a > 0), ou máximo (se a< 0);
       4. A reta que passa por V e é paralela ao eixo dos y é o eixo de simetria da parábola;
       5. Para x = 0 , temos y = a · 02 + b · 0 + c = c; então (0, c) é o ponto em que a parábola
          corta o eixo dos y.
Sinal
  Consideramos uma função quadrática y = f(x) = ax2 + bx + c e determinemos os valores de
x para os quais y é negativo e os valores de x para os quais y é positivos.
   Conforme o sinal do discriminante = b2 - 4ac, podemos ocorrer os seguintes casos:
1º - > 0
  Nesse caso a função quadrática admite dois zeros reais distintos (x1 x2). a parábola
intercepta o eixo Ox em dois pontos e o sinal da função é o indicado nos gráficos abaixo:




                   quando a > 0                                    quando a < 0


              y>0   (x < x1 ou x > x2)                         y>0     x1 < x < x2
                y<0     x1 < x < x2                          y<0   (x < x1 ou x > x2)




2º -     =0




                                                                                                52
quando a > 0                           quando a < 0




3º -   <0




            quando a > 0                           quando a < 0




                           EQUAÇÕES EXPONENCIAIS

                                                                  53
Chamamos de equações exponenciais toda equação na qual a incógnita aparece
em expoente.

Exemplos de equações exponenciais:
1) 3x =81 (a solução é x=4)
2) 2x-5=16 (a solução é x=9)
3) 16x-42x-1-10=22x-1 (a solução é x=1)
4) 32x-1-3x-3x-1+1=0 (as soluções são x’=0 e x’’=1)

   Para resolver equações exponenciais, devemos realizar dois passos importantes:
   1º) redução dos dois membros da equação a potências de mesma base;
   2º) aplicação da propriedade:

                    a m = a n ⇒ m = n (a ≠ 1 e a > 0)


EXERCÍCIOS RESOLVIDOS:

1) 3x=81
Resolução: Como 81=34, podemos escrever 3x = 34
E daí, x=4.

2) 9x = 1
Resolução: 9x = 1 ⇒ 9x = 90 ; logo x=0.

       x
   3    81
3)   =
   4   256
                    x              x              x       4
            3    81   3  34   3   3
Resolução :   =     ⇒   = 4 ⇒   =   ; então x = 4.
            4   256   4  4    4   4


4) 3 x = 4 27
                                              3
                                                          3
Resolução : 3 = 27 ⇒ 3 = 3 ⇒ 3 = 3 ; logo x =
                x   4      x   4   3      x   4
                                                          4
5) 23x-1 = 322x
Resolução: 23x-1 = 322x ⇒ 23x-1 = (25)2x ⇒ 23x-1 = 210x ; daí 3x-1=10,
de onde x=-1/7.




6) Resolva a equação 32x–6.3x–27=0.
                                                                                    54
Resolução: vamos resolver esta equação através de uma transformação:
32x–6.3x–27=0 ⇒ (3x)2-6.3x–27=0
Fazendo 3x=y, obtemos:
y2-6y–27=0 ; aplicando Bhaskara encontramos ⇒ y’=-3 e y’’=9
Para achar o x, devemos voltar os valores para a equação auxiliar 3x=y:

y’=-3 ⇒ 3x’ = -3 ⇒ não existe x’, pois potência de base positiva é positiva
y’’=9 ⇒ 3x’’ = 9 ⇒ 3x’’ = 32 ⇒ x’’=2

Portanto a solução é x=2


                                 FUNÇÃO EXPONENCIAL

      Chamamos de funções exponenciais aquelas nas quais temos a variável
aparecendo em expoente.
      A função f:IRIR+ definida por f(x)=ax, com a ∈ IR+ e a≠1, é chamada função
exponencial de base a. O domínio dessa função é o conjunto IR (reais) e o
contradomínio é IR+ (reais positivos, maiores que zero).


                GRÁFICO CARTESIANO DA FUNÇÃO EXPONENCIAL

      Temos 2 casos a considerar:
       quando a>1;
       quando 0<a<1.

      Acompanhe os exemplos seguintes:

1) y=2x (nesse caso, a=2, logo a>1)
   Atribuindo alguns valores a x e calculando os correspondentes valores de y, obtemos
   a tabela e o gráfico abaixo:



                   x       -2         -1       0        1        2
                   y       1/4        1/2      1        2        4



2) y=(1/2)x (nesse caso, a=1/2, logo 0<a<1)
   Atribuindo alguns valores a x e calculando os correspondentes valores de y, obtemos
   a tabela e o gráfico abaixo:




                                                                                   55
x       -2       -1         0        1        2
                 y       4        2          1       1/2      1/4




  Nos dois exemplos, podemos observar que
  a) o gráfico nunca intercepta o eixo horizontal; a função não tem raízes;
  b) o gráfico corta o eixo vertical no ponto (0,1);
  c) os valores de y são sempre positivos (potência de base positiva é positiva),
     portanto o conjunto imagem é Im=IR+.

  Além disso, podemos estabelecer o seguinte:



               a>1                                  0<a<1




      f(x) é crescente e Im=IR+            f(x) é decrescente e Im=IR+
 Para quaisquer x1 e x2 do domínio:     Para quaisquer x1 e x2 do domínio:
x2>x1 ⇒ y2>y1 (as desigualdades têm    x2>x1 ⇒ y2<y1 (as desigualdades têm
           mesmo sentido)                      sentidos diferentes)




                          INEQUAÇÕES EXPONENCIAIS

      Chamamos de inequações exponenciais toda inequação na qual a incógnita
aparece em expoente.


                                                                               56
Exemplos de inequações exponenciais:

1) 3 x > 81 (a solução é x > 4)
                   2
                       −1
2) 2 2x -2 ≤ 2 x                 (que é satisfeita para todo x real)
          x                 −3
   4      4
3)   ≥           (que é satisfeita para x ≤ -3)
   5      5
4) 25 x - 150.5 x + 3125 < 0 (que é satisfeita para 2 < x < 3)


    Para resolver inequações exponenciais, devemos realizar dois passos importantes:
    1º) redução dos dois membros da inequação a potências de mesma base;
    2º) aplicação da propriedade:

                                 a>1                                       0<a<1
              a > a ⇒ m>n
              m                  n
                                                                   a > an ⇒ m<n
                                                                       m

(as desigualdades têm mesmo sentido)                         (as desigualdades têm sentidos
                                                                       diferentes)


EXERCÍCIO RESOLVIDO:

                                     − 11
1) 4 x −1 + 4 x − 4 x +1 >
                                      4
Resolução :
                             4x                  − 11
A inequação pode ser escrita    + 4 x − 4 x .4 >      .
                              4                   4
Multiplicando ambos os lados por 4 temos :
4 x + 4.4 x − 16.4 x > −11 , ou seja :
(1 + 4 − 16).4 x > −11 ⇒ -11.4 x > −11 e daí, 4 x < 1
Porém, 4 x < 1 ⇒ 4 x < 4 0.
Como a base (4) é maior que 1, obtemos :
4 x < 40 ⇒ x < 0
Portanto S = IR - (reais negativos)



                                               FUNÇÃO LOGARÍTMICA

      A função f:IR+IR definida por f(x)=logax, com a≠1 e a>0, é chamada função
logarítmica de base a. O domínio dessa função é o conjunto IR+ (reais positivos,
maiores que zero) e o contradomínio é IR (reais).
                                                                                              57
GRÁFICO CARTESIANO DA FUNÇÃO LOGARÍTMICA

        Temos 2 casos a considerar:
         quando a>1;
         quando 0<a<1.

        Acompanhe nos exemplos seguintes, a construção do gráfico em cada caso:

3)   y=log2x (nesse caso, a=2, logo a>1)
     Atribuindo alguns valores a x e calculando os correspondentes valores de y, obtemos
     a tabela e o gráfico abaixo:


                    x       1/4       1/2       1        2        4
                    y       -2        -1        0        1        2




4)   y=log(1/2)x (nesse caso, a=1/2, logo 0<a<1)
     Atribuindo alguns valores a x e calculando os correspondentes valores de y, obtemos
     a tabela e o gráfico abaixo:



                    x       1/4       1/2       1        2        4
                    y        2         1        0        -1       -2




                                                                                     58
Nos dois exemplos, podemos observar que
  d) o gráfico nunca intercepta o eixo vertical;
  e) o gráfico corta o eixo horizontal no ponto (1,0). A raiz da função é x=1;
  f) y assume todos os valores reais, portanto o conjunto imagem é Im=IR.

  Além disso, podemos estabelecer o seguinte:

               a>1                                    0<a<1




      f(x) é crescente e Im=IR               f(x) é decrescente e Im=IR
 Para quaisquer x1 e x2 do domínio:      Para quaisquer x1 e x2 do domínio:
x2>x1 ⇒ y2>y1 (as desigualdades têm     x2>x1 ⇒ y2<y1 (as desigualdades têm
           mesmo sentido)                        sentidos diferentes)




                                                                                 59
EQUAÇÕES LOGARÍTMICAS

      Chamamos de equações logarítmicas toda equação que envolve logaritmos com
a incógnita aparecendo no logaritmando, na base ou em ambos.

Exemplos de equações logarítmicas:
7)  log3x =5 (a solução é x=243)
          2
8) log(x -1) = log 3 (as soluções são x’=-2 e x’’=2)
9) log2(x+3) + log2(x-3) = log27 (a solução é x=4)
            2
10) logx+1(x -x)=2 (a solução é x=-1/3)

Alguns exemplos resolvidos:
1)log3(x+5) = 2
  Resolução: condição de existência: x+5>0 => x>-5
  log3(x+5) = 2 => x+5 = 32 => x=9-5 => x=4
  Como x=4 satisfaz a condição de existência, então o conjunto solução é
S={4}.

2) log2(log4 x) = 1
   Resolução: condição de existência: x>0 e log4x>0
   log2(log4 x) = 1 ; sabemos que 1 = log2(2), então
   log2(log4x) = log2(2) => log4x = 2 => 42 = x => x=16
   Como x=16 satisfaz as condições de existência, então o conjunto solução
é S={16}.

3) Resolva o sistema:
log x + log y = 7

3. log x − 2. log y = 1

Resolução: condições de existência: x>0 e y>0
Da primeira equação temos:
log x+log y=7 => log y = 7-log x
Substituindo log y na segunda equação temos:
3.log x – 2.(7-log x)=1 => 3.log x-14+2.log x = 1 => 5.log x = 15 =>

=> log x =3 => x=103
Substituindo x= 103 em log y = 7-log x temos:
log y = 7- log 103 => log y = 7-3 => log y =4 => y=104.
Como essas raízes satisfazem as condições de existência, então o conjunto solução é
S={(103;104)}.


                                                                                      60
INEQUAÇÕES LOGARÍTMICAS

     Chamamos de inequações logarítmicas toda inequação que envolve logaritmos
com a incógnita aparecendo no logaritmando, na base ou em ambos.

Exemplos de inequações logarítmicas:
  1) log2x > 0 (a solução é x>1)
  2) log4(x+3) ≤ 1 (a solução é –3<x≤1)

   Para resolver inequações logarítmicas, devemos realizar dois passos importantes:
   1º) redução dos dois membros da inequação a logaritmos de mesma base;
   2º) aplicação da propriedade:

                 a>1                                    0<a<1
   logam > logan ⇒ m>n>0                   logam > logan ⇒ 0<m<n
(as desigualdades têm mesmo sentido)        (as desigualdades têm sentidos
                                                      diferentes)

EXERCÍCIOS RESOLVIDOS:

1) log2(x+2) > log28
   Resolução:
   Condições de existência: x+2>0, ou seja, x>-2 (S1)
   Como a base (2) é maior que 1, temos:
   x+2>8 e, daí, x>6 (S2)
   O conjunto solução é S= S1 ∩ S2 = {x ∈ IR| x>6}.
   Portanto a solução final é a intersecção de S1 e S2, como está representado logo
abaixo no desenho:




2) log2(log3x) ≥ 0
   Resolução:
   Condições de existência: x>0 e log3x>0
   Como log21=0, a inequação pode ser escrita assim:
   log2(log3x) ≥ log21
   Sendo a base (2) maior que 1, temos: log3x ≥ 1.
   Como log33 = 1, então, log3x ≥ log33 e, daí, x ≥ 3, porque a base (3) é maior que 1.
   As condições de existência estão satisfeitas, portanto S={x ∈ IR| x ≥ 3}.


                                                                                          61
FUNÇÃO MODULAR

•   Módulo (ou valor absoluto) de um número

   O módulo (ou valor absoluto) de um número real x, que se indica por | x | é definido da seguinte
maneira:
                                     x, se x ≥ 0
                                 x =
                                    − x, se x < 0

    Então:
     se x é positivo ou zero, | x | é igual ao próprio x.
       Exemplos: | 2 | = 2 ; | 1/2 | = | 1/2 | ; | 15 | = 15

     se x é negativo, | x | é igual a -x.
       Exemplos: | -2 | = -(-2) = 2 ; | -20 | = -(-20) = 20

    O módulo de um número real é sempre positivo ou nulo. O módulo de um número real nunca é
negativo.
    Representando geometricamente, o módulo de um número real x é igual a distância do ponto que
representa, na reta real, o número x ao ponto 0 de origem. Assim:

    •   Se | x | < a (com a>0) significa que a distância entre x e a origem é menor que a, isto é, x deve
        estar entre –a e a, ou seja, | x | < a ⇔ -a < x < a.




    •   Se | x | > a (com a>0) significa que a distância entre x e a origem é maior que a, isto é, deve
        estar à direita de a ou à esquerda de –a na reta real, ou seja: | x | > a ⇔ x > a ou x < -a.




•   Equações modulares

  Toda a equação que contiver a incógnita em um módulo num dos membros será chamada equação
modular.

    Exemplos:



    a) | x2-5x | = 1
    b) | x+8 | = | x2-3 |




                                                                                                      62
ALGUMAS EQUAÇÕES MODULARES RESOLVIDAS:

   1) Resolver a equação | x2-5x | = 6.
      Resolução: Temos que analisar dois casos:
             caso 1: x2-5x = 6
             caso 2: x2-5x = -6

            Resolvendo o caso 1:
            x2-5x-6 = 0 => x’=6 e x’’=-1.
            Resolvendo o caso 2:
            x2-5x+6 = 0 => x’=3 e x’’=2.
      Resposta: S={-1,2,3,6}


   2) Resolver a equação | x-6 | = | 3-2x |.
      Resolução: Temos que analisar dois casos:
             caso 1: x-6 = 3-2x
             caso 2: x-6 = -(3-2x)
             Resolvendo o caso 1:
             x-6 = 3-2x => x+2x = 3+6 => 3x=9 => x=3
             Resolvendo o caso 2:
             x-6 = -(3-2x) => x-2x = -3+6 => -x=3 => x=-3
      Resposta: S={-3,3}


• Inequações modulares

   Chamamos de inequações modulares as inequações nos quais aparecem módulos de expressões que
contém a incógnita.

   Algumas inequações modulares resolvidas:

   1) Resolver a inequação | -2x+6 | < 2.
      Resolução:
                                                − 2 < −2 x + 6   2 x < 6 + 2
       | - 2x + 6 | < 2 ⇒ − 2 < −2 x + 6 < 2 ⇒                 ⇒             ⇒
                                               − 2 x + 6 < 2     − 2 x < 4
            2 x < 8     x < 4
       ⇒              ⇒ 
            2 x > 4     x > 2

      S = {x ∈ IR | 2<x<4}




                                                                                           63
2) Dê o conjunto solução da inequação |x2-2x+3| ≤ 4.

    Resolução:
    |x2-2x+3| ≤ 4 => -4 ≤ x2-2x+3 ≤ 4.
    Então temos duais inequações (que devem ser satisfeitas ao mesmo tempo):
    Eq.1: -4 ≤ x2-2x+3
    Eq.2: x2-2x+3 ≤ 4

    Resolvendo a Eq.1:
    -4 ≤ x2-2x+3 => -4-3 ≤ x2-2x => -7 ≤ x2-2x => x2-2x+7 ≥ 0 => sem raízes reais

    Resolvendo a Eq.2:
    x2-2x+3 ≤ 4 => x2-2x-1 ≤ 0

                                              x' = 1 − 2
                                             
    Aplicando Bhaskara encontramos as raízes 
                                              x' ' = 1 + 2
                                             
    S = {x ∈ IR | 1 − 2 ≤ x ≤ 1 + 2}


•   Módulo e raiz quadrada

    Consideremos os números reais x e y.
    Temos por definição, que
       x=y
    se e somente se, y2 = x e y≥0. Daí podemos concluir que
       x2 = x
    só é verdadeiro se x≥0.
    Se tivermos x<0, não podemos afirmar que
     x2 = x
    pois isso contradiz a definição.

    Por exemplo, se x=-3, teríamos:
     ( −3) 2 = −3
    o que é um absurdo, pois o primeiro membro é positivo e o segundo negativo. Usando a definição
    de módulo, podemos escrever:

       x 2 =| x |
    o que é verdadeiro para todo x real.


    Devemos proceder da mesma forma em relação a todas raízes de índice par:
    4
        x 4 =| x |,       6
                              x 6 =| x |,    2n
                                                  x 2 n =| x |, com x ∈ IR e n ∈ IN *

    Com relação às raízes de índice ímpar, podemos escrever:
                                        2 n +1
    3
        x 3 = x,      5
                          x 5 = x,               x 2 n+1 = x, com x ∈ IR e n ∈ IN


                                                                                               64
Análise Combinatória
Análise Combinatória
Análise Combinatória
Análise Combinatória
Análise Combinatória
Análise Combinatória
Análise Combinatória
Análise Combinatória
Análise Combinatória
Análise Combinatória
Análise Combinatória
Análise Combinatória
Análise Combinatória
Análise Combinatória
Análise Combinatória
Análise Combinatória
Análise Combinatória
Análise Combinatória
Análise Combinatória
Análise Combinatória
Análise Combinatória
Análise Combinatória
Análise Combinatória
Análise Combinatória
Análise Combinatória
Análise Combinatória
Análise Combinatória
Análise Combinatória
Análise Combinatória
Análise Combinatória
Análise Combinatória
Análise Combinatória
Análise Combinatória
Análise Combinatória
Análise Combinatória
Análise Combinatória
Análise Combinatória
Análise Combinatória
Análise Combinatória
Análise Combinatória
Análise Combinatória
Análise Combinatória
Análise Combinatória
Análise Combinatória
Análise Combinatória
Análise Combinatória
Análise Combinatória
Análise Combinatória
Análise Combinatória
Análise Combinatória
Análise Combinatória
Análise Combinatória
Análise Combinatória
Análise Combinatória
Análise Combinatória
Análise Combinatória
Análise Combinatória
Análise Combinatória
Análise Combinatória
Análise Combinatória
Análise Combinatória

Mais conteúdo relacionado

Semelhante a Análise Combinatória

Matematica basica
Matematica basicaMatematica basica
Matematica basicaaletriak
 
Análise combinatória (resumo e exercícios)
Análise combinatória (resumo e exercícios)Análise combinatória (resumo e exercícios)
Análise combinatória (resumo e exercícios)josivaldopassos
 
Análise combinatória (resumo)
Análise combinatória (resumo)Análise combinatória (resumo)
Análise combinatória (resumo)josivaldopassos
 
Análise combinatória (resumo)
Análise combinatória (resumo)Análise combinatória (resumo)
Análise combinatória (resumo)josivaldopassos
 
Analise combinatoria 1
Analise combinatoria 1Analise combinatoria 1
Analise combinatoria 1SEDUC-PA
 
Analise combinatoria
Analise combinatoriaAnalise combinatoria
Analise combinatoriaValter Carlos
 
Apostila análise combinatória
Apostila   análise combinatóriaApostila   análise combinatória
Apostila análise combinatóriaMaria Alves
 
Apostila análise combinatória
Apostila   análise combinatóriaApostila   análise combinatória
Apostila análise combinatóriaMaria Alves
 
Analise Comb E Probabilidades
Analise Comb E ProbabilidadesAnalise Comb E Probabilidades
Analise Comb E Probabilidadesgueste0e57c
 
Analise Comb E Probabilidades
Analise Comb E ProbabilidadesAnalise Comb E Probabilidades
Analise Comb E ProbabilidadesISJ
 
Análise combinatória (resumo e exercícios)
Análise combinatória (resumo e exercícios)Análise combinatória (resumo e exercícios)
Análise combinatória (resumo e exercícios)josivaldopassos
 
Caderno matematica
Caderno matematicaCaderno matematica
Caderno matematicafazag
 
7ª SéRie MatemáTica 1º Semestre
7ª SéRie   MatemáTica   1º Semestre7ª SéRie   MatemáTica   1º Semestre
7ª SéRie MatemáTica 1º SemestrePROFESSOR FABRÍCIO
 
7ª SéRie MatemáTica 1º Semestre
7ª SéRie   MatemáTica   1º Semestre7ª SéRie   MatemáTica   1º Semestre
7ª SéRie MatemáTica 1º SemestrePROFESSOR FABRÍCIO
 
Implementação modulo3
Implementação modulo3Implementação modulo3
Implementação modulo3inechidias
 
Implementação modulo3
Implementação modulo3Implementação modulo3
Implementação modulo3inechidias
 

Semelhante a Análise Combinatória (20)

Matematica basica
Matematica basicaMatematica basica
Matematica basica
 
ANÁLISE COMBINATÓRIA
ANÁLISE COMBINATÓRIA ANÁLISE COMBINATÓRIA
ANÁLISE COMBINATÓRIA
 
Apostila Professor Linhares
Apostila Professor LinharesApostila Professor Linhares
Apostila Professor Linhares
 
Apostila professor Linhares
Apostila professor LinharesApostila professor Linhares
Apostila professor Linhares
 
Análise combinatória (resumo e exercícios)
Análise combinatória (resumo e exercícios)Análise combinatória (resumo e exercícios)
Análise combinatória (resumo e exercícios)
 
Análise combinatória (resumo)
Análise combinatória (resumo)Análise combinatória (resumo)
Análise combinatória (resumo)
 
Análise combinatória (resumo)
Análise combinatória (resumo)Análise combinatória (resumo)
Análise combinatória (resumo)
 
Analise combinatoria 1
Analise combinatoria 1Analise combinatoria 1
Analise combinatoria 1
 
Analise combinatoria
Analise combinatoriaAnalise combinatoria
Analise combinatoria
 
Análise combinatória
Análise  combinatóriaAnálise  combinatória
Análise combinatória
 
Apostila análise combinatória
Apostila   análise combinatóriaApostila   análise combinatória
Apostila análise combinatória
 
Apostila análise combinatória
Apostila   análise combinatóriaApostila   análise combinatória
Apostila análise combinatória
 
Analise Comb E Probabilidades
Analise Comb E ProbabilidadesAnalise Comb E Probabilidades
Analise Comb E Probabilidades
 
Analise Comb E Probabilidades
Analise Comb E ProbabilidadesAnalise Comb E Probabilidades
Analise Comb E Probabilidades
 
Análise combinatória (resumo e exercícios)
Análise combinatória (resumo e exercícios)Análise combinatória (resumo e exercícios)
Análise combinatória (resumo e exercícios)
 
Caderno matematica
Caderno matematicaCaderno matematica
Caderno matematica
 
7ª SéRie MatemáTica 1º Semestre
7ª SéRie   MatemáTica   1º Semestre7ª SéRie   MatemáTica   1º Semestre
7ª SéRie MatemáTica 1º Semestre
 
7ª SéRie MatemáTica 1º Semestre
7ª SéRie   MatemáTica   1º Semestre7ª SéRie   MatemáTica   1º Semestre
7ª SéRie MatemáTica 1º Semestre
 
Implementação modulo3
Implementação modulo3Implementação modulo3
Implementação modulo3
 
Implementação modulo3
Implementação modulo3Implementação modulo3
Implementação modulo3
 

Mais de manjeri_k

Massa multiprática pastel assado
Massa multiprática  pastel assadoMassa multiprática  pastel assado
Massa multiprática pastel assadomanjeri_k
 
Flã e mousses
Flã e moussesFlã e mousses
Flã e moussesmanjeri_k
 
Bolo nega maluca
Bolo nega malucaBolo nega maluca
Bolo nega malucamanjeri_k
 
Bolo de maracuja
Bolo de maracujaBolo de maracuja
Bolo de maracujamanjeri_k
 
Bolo de canela
Bolo de canelaBolo de canela
Bolo de canelamanjeri_k
 
Aprendaafazersalgados receitas
Aprendaafazersalgados receitasAprendaafazersalgados receitas
Aprendaafazersalgados receitasmanjeri_k
 

Mais de manjeri_k (7)

Massa multiprática pastel assado
Massa multiprática  pastel assadoMassa multiprática  pastel assado
Massa multiprática pastel assado
 
Flã e mousses
Flã e moussesFlã e mousses
Flã e mousses
 
Bolo nega maluca
Bolo nega malucaBolo nega maluca
Bolo nega maluca
 
Bolo de maracuja
Bolo de maracujaBolo de maracuja
Bolo de maracuja
 
Bolo de canela
Bolo de canelaBolo de canela
Bolo de canela
 
Aprendaafazersalgados receitas
Aprendaafazersalgados receitasAprendaafazersalgados receitas
Aprendaafazersalgados receitas
 
Sobremesas
SobremesasSobremesas
Sobremesas
 

Último

aula de bioquímica bioquímica dos carboidratos.ppt
aula de bioquímica bioquímica dos carboidratos.pptaula de bioquímica bioquímica dos carboidratos.ppt
aula de bioquímica bioquímica dos carboidratos.pptssuser2b53fe
 
Nós Propomos! " Pinhais limpos, mundo saudável"
Nós Propomos! " Pinhais limpos, mundo saudável"Nós Propomos! " Pinhais limpos, mundo saudável"
Nós Propomos! " Pinhais limpos, mundo saudável"Ilda Bicacro
 
Revolução russa e mexicana. Slides explicativos e atividades
Revolução russa e mexicana. Slides explicativos e atividadesRevolução russa e mexicana. Slides explicativos e atividades
Revolução russa e mexicana. Slides explicativos e atividadesFabianeMartins35
 
PROJETO DE EXTENSÃO I - Radiologia Tecnologia
PROJETO DE EXTENSÃO I - Radiologia TecnologiaPROJETO DE EXTENSÃO I - Radiologia Tecnologia
PROJETO DE EXTENSÃO I - Radiologia TecnologiaHELENO FAVACHO
 
Slides Lição 6, CPAD, As Nossas Armas Espirituais, 2Tr24.pptx
Slides Lição 6, CPAD, As Nossas Armas Espirituais, 2Tr24.pptxSlides Lição 6, CPAD, As Nossas Armas Espirituais, 2Tr24.pptx
Slides Lição 6, CPAD, As Nossas Armas Espirituais, 2Tr24.pptxLuizHenriquedeAlmeid6
 
"É melhor praticar para a nota" - Como avaliar comportamentos em contextos de...
"É melhor praticar para a nota" - Como avaliar comportamentos em contextos de..."É melhor praticar para a nota" - Como avaliar comportamentos em contextos de...
"É melhor praticar para a nota" - Como avaliar comportamentos em contextos de...Rosalina Simão Nunes
 
Projeto_de_Extensão_Agronomia_adquira_ja_(91)_98764-0830.pdf
Projeto_de_Extensão_Agronomia_adquira_ja_(91)_98764-0830.pdfProjeto_de_Extensão_Agronomia_adquira_ja_(91)_98764-0830.pdf
Projeto_de_Extensão_Agronomia_adquira_ja_(91)_98764-0830.pdfHELENO FAVACHO
 
Jogo de Rimas - Para impressão em pdf a ser usado para crianças
Jogo de Rimas - Para impressão em pdf a ser usado para criançasJogo de Rimas - Para impressão em pdf a ser usado para crianças
Jogo de Rimas - Para impressão em pdf a ser usado para criançasSocorro Machado
 
PROJETO DE EXTENSÃO I - TERAPIAS INTEGRATIVAS E COMPLEMENTARES.pdf
PROJETO DE EXTENSÃO I - TERAPIAS INTEGRATIVAS E COMPLEMENTARES.pdfPROJETO DE EXTENSÃO I - TERAPIAS INTEGRATIVAS E COMPLEMENTARES.pdf
PROJETO DE EXTENSÃO I - TERAPIAS INTEGRATIVAS E COMPLEMENTARES.pdfHELENO FAVACHO
 
PROVA - ESTUDO CONTEMPORÂNEO E TRANSVERSAL: LEITURA DE IMAGENS, GRÁFICOS E MA...
PROVA - ESTUDO CONTEMPORÂNEO E TRANSVERSAL: LEITURA DE IMAGENS, GRÁFICOS E MA...PROVA - ESTUDO CONTEMPORÂNEO E TRANSVERSAL: LEITURA DE IMAGENS, GRÁFICOS E MA...
PROVA - ESTUDO CONTEMPORÂNEO E TRANSVERSAL: LEITURA DE IMAGENS, GRÁFICOS E MA...azulassessoria9
 
PRÁTICAS PEDAGÓGICAS GESTÃO DA APRENDIZAGEM
PRÁTICAS PEDAGÓGICAS GESTÃO DA APRENDIZAGEMPRÁTICAS PEDAGÓGICAS GESTÃO DA APRENDIZAGEM
PRÁTICAS PEDAGÓGICAS GESTÃO DA APRENDIZAGEMHELENO FAVACHO
 
PROJETO DE EXTENSÃO - EDUCAÇÃO FÍSICA BACHARELADO.pdf
PROJETO DE EXTENSÃO - EDUCAÇÃO FÍSICA BACHARELADO.pdfPROJETO DE EXTENSÃO - EDUCAÇÃO FÍSICA BACHARELADO.pdf
PROJETO DE EXTENSÃO - EDUCAÇÃO FÍSICA BACHARELADO.pdfHELENO FAVACHO
 
matematica aula didatica prática e tecni
matematica aula didatica prática e tecnimatematica aula didatica prática e tecni
matematica aula didatica prática e tecniCleidianeCarvalhoPer
 
PROJETO DE EXTENÇÃO - GESTÃO DE RECURSOS HUMANOS.pdf
PROJETO DE EXTENÇÃO - GESTÃO DE RECURSOS HUMANOS.pdfPROJETO DE EXTENÇÃO - GESTÃO DE RECURSOS HUMANOS.pdf
PROJETO DE EXTENÇÃO - GESTÃO DE RECURSOS HUMANOS.pdfHELENO FAVACHO
 
planejamento_estrategico_-_gestao_2021-2024_16015654.pdf
planejamento_estrategico_-_gestao_2021-2024_16015654.pdfplanejamento_estrategico_-_gestao_2021-2024_16015654.pdf
planejamento_estrategico_-_gestao_2021-2024_16015654.pdfmaurocesarpaesalmeid
 
COMPETÊNCIA 2 da redação do enem prodção textual professora vanessa cavalcante
COMPETÊNCIA 2 da redação do enem prodção textual professora vanessa cavalcanteCOMPETÊNCIA 2 da redação do enem prodção textual professora vanessa cavalcante
COMPETÊNCIA 2 da redação do enem prodção textual professora vanessa cavalcanteVanessaCavalcante37
 
About Vila Galé- Cadeia Empresarial de Hotéis
About Vila Galé- Cadeia Empresarial de HotéisAbout Vila Galé- Cadeia Empresarial de Hotéis
About Vila Galé- Cadeia Empresarial de Hotéisines09cachapa
 
apostila projeto de vida 2 ano ensino médio
apostila projeto de vida 2 ano ensino médioapostila projeto de vida 2 ano ensino médio
apostila projeto de vida 2 ano ensino médiorosenilrucks
 
PRÉDIOS HISTÓRICOS DE ASSARÉ Prof. Francisco Leite.pdf
PRÉDIOS HISTÓRICOS DE ASSARÉ Prof. Francisco Leite.pdfPRÉDIOS HISTÓRICOS DE ASSARÉ Prof. Francisco Leite.pdf
PRÉDIOS HISTÓRICOS DE ASSARÉ Prof. Francisco Leite.pdfprofesfrancleite
 

Último (20)

aula de bioquímica bioquímica dos carboidratos.ppt
aula de bioquímica bioquímica dos carboidratos.pptaula de bioquímica bioquímica dos carboidratos.ppt
aula de bioquímica bioquímica dos carboidratos.ppt
 
Nós Propomos! " Pinhais limpos, mundo saudável"
Nós Propomos! " Pinhais limpos, mundo saudável"Nós Propomos! " Pinhais limpos, mundo saudável"
Nós Propomos! " Pinhais limpos, mundo saudável"
 
Revolução russa e mexicana. Slides explicativos e atividades
Revolução russa e mexicana. Slides explicativos e atividadesRevolução russa e mexicana. Slides explicativos e atividades
Revolução russa e mexicana. Slides explicativos e atividades
 
PROJETO DE EXTENSÃO I - Radiologia Tecnologia
PROJETO DE EXTENSÃO I - Radiologia TecnologiaPROJETO DE EXTENSÃO I - Radiologia Tecnologia
PROJETO DE EXTENSÃO I - Radiologia Tecnologia
 
Slides Lição 6, CPAD, As Nossas Armas Espirituais, 2Tr24.pptx
Slides Lição 6, CPAD, As Nossas Armas Espirituais, 2Tr24.pptxSlides Lição 6, CPAD, As Nossas Armas Espirituais, 2Tr24.pptx
Slides Lição 6, CPAD, As Nossas Armas Espirituais, 2Tr24.pptx
 
"É melhor praticar para a nota" - Como avaliar comportamentos em contextos de...
"É melhor praticar para a nota" - Como avaliar comportamentos em contextos de..."É melhor praticar para a nota" - Como avaliar comportamentos em contextos de...
"É melhor praticar para a nota" - Como avaliar comportamentos em contextos de...
 
Projeto_de_Extensão_Agronomia_adquira_ja_(91)_98764-0830.pdf
Projeto_de_Extensão_Agronomia_adquira_ja_(91)_98764-0830.pdfProjeto_de_Extensão_Agronomia_adquira_ja_(91)_98764-0830.pdf
Projeto_de_Extensão_Agronomia_adquira_ja_(91)_98764-0830.pdf
 
Aula sobre o Imperialismo Europeu no século XIX
Aula sobre o Imperialismo Europeu no século XIXAula sobre o Imperialismo Europeu no século XIX
Aula sobre o Imperialismo Europeu no século XIX
 
Jogo de Rimas - Para impressão em pdf a ser usado para crianças
Jogo de Rimas - Para impressão em pdf a ser usado para criançasJogo de Rimas - Para impressão em pdf a ser usado para crianças
Jogo de Rimas - Para impressão em pdf a ser usado para crianças
 
PROJETO DE EXTENSÃO I - TERAPIAS INTEGRATIVAS E COMPLEMENTARES.pdf
PROJETO DE EXTENSÃO I - TERAPIAS INTEGRATIVAS E COMPLEMENTARES.pdfPROJETO DE EXTENSÃO I - TERAPIAS INTEGRATIVAS E COMPLEMENTARES.pdf
PROJETO DE EXTENSÃO I - TERAPIAS INTEGRATIVAS E COMPLEMENTARES.pdf
 
PROVA - ESTUDO CONTEMPORÂNEO E TRANSVERSAL: LEITURA DE IMAGENS, GRÁFICOS E MA...
PROVA - ESTUDO CONTEMPORÂNEO E TRANSVERSAL: LEITURA DE IMAGENS, GRÁFICOS E MA...PROVA - ESTUDO CONTEMPORÂNEO E TRANSVERSAL: LEITURA DE IMAGENS, GRÁFICOS E MA...
PROVA - ESTUDO CONTEMPORÂNEO E TRANSVERSAL: LEITURA DE IMAGENS, GRÁFICOS E MA...
 
PRÁTICAS PEDAGÓGICAS GESTÃO DA APRENDIZAGEM
PRÁTICAS PEDAGÓGICAS GESTÃO DA APRENDIZAGEMPRÁTICAS PEDAGÓGICAS GESTÃO DA APRENDIZAGEM
PRÁTICAS PEDAGÓGICAS GESTÃO DA APRENDIZAGEM
 
PROJETO DE EXTENSÃO - EDUCAÇÃO FÍSICA BACHARELADO.pdf
PROJETO DE EXTENSÃO - EDUCAÇÃO FÍSICA BACHARELADO.pdfPROJETO DE EXTENSÃO - EDUCAÇÃO FÍSICA BACHARELADO.pdf
PROJETO DE EXTENSÃO - EDUCAÇÃO FÍSICA BACHARELADO.pdf
 
matematica aula didatica prática e tecni
matematica aula didatica prática e tecnimatematica aula didatica prática e tecni
matematica aula didatica prática e tecni
 
PROJETO DE EXTENÇÃO - GESTÃO DE RECURSOS HUMANOS.pdf
PROJETO DE EXTENÇÃO - GESTÃO DE RECURSOS HUMANOS.pdfPROJETO DE EXTENÇÃO - GESTÃO DE RECURSOS HUMANOS.pdf
PROJETO DE EXTENÇÃO - GESTÃO DE RECURSOS HUMANOS.pdf
 
planejamento_estrategico_-_gestao_2021-2024_16015654.pdf
planejamento_estrategico_-_gestao_2021-2024_16015654.pdfplanejamento_estrategico_-_gestao_2021-2024_16015654.pdf
planejamento_estrategico_-_gestao_2021-2024_16015654.pdf
 
COMPETÊNCIA 2 da redação do enem prodção textual professora vanessa cavalcante
COMPETÊNCIA 2 da redação do enem prodção textual professora vanessa cavalcanteCOMPETÊNCIA 2 da redação do enem prodção textual professora vanessa cavalcante
COMPETÊNCIA 2 da redação do enem prodção textual professora vanessa cavalcante
 
About Vila Galé- Cadeia Empresarial de Hotéis
About Vila Galé- Cadeia Empresarial de HotéisAbout Vila Galé- Cadeia Empresarial de Hotéis
About Vila Galé- Cadeia Empresarial de Hotéis
 
apostila projeto de vida 2 ano ensino médio
apostila projeto de vida 2 ano ensino médioapostila projeto de vida 2 ano ensino médio
apostila projeto de vida 2 ano ensino médio
 
PRÉDIOS HISTÓRICOS DE ASSARÉ Prof. Francisco Leite.pdf
PRÉDIOS HISTÓRICOS DE ASSARÉ Prof. Francisco Leite.pdfPRÉDIOS HISTÓRICOS DE ASSARÉ Prof. Francisco Leite.pdf
PRÉDIOS HISTÓRICOS DE ASSARÉ Prof. Francisco Leite.pdf
 

Análise Combinatória

  • 1. Análise Combinatória Fatorial de um número: n!=n.(n-1).(n-2)...3.2.1 Definições especiais: 0!=1 1!=1 100!+101! 1) Calcule o valor da expressão . 99! 100!+101! 100.99!+101.100.99! = = 100 + 101.100 = 100 + 10100 = 10200 99! 99! ( x + 1)! 2) Resolva a equação = 56. ( x − 1)! ( x + 1)! ( x + 1)( x)( x − 1)! = 56 ⇒ = 56 ⇒ ( x + 1)( x) = 56 ⇒ x 2 + x = 56 ⇒ ( x − 1)! ( x − 1)! − 1 ± 225 − 1 ± 15 x = 7 ⇒ x 2 + x − 56 = 0 ⇒ x = ⇒ x= ⇒ 2 2 x = -8 Resposta : x = 7, pois não existe fatorial de um número negativo. 3) Quatro times de futebol (Grêmio, Santos, São Paulo e Flamengo) disputam o torneio dos campeões do mundo. Quantas são as possibilidades para os três primeiros lugares? R : Existem 4 possibilidades para o 1º lugar, sobrando 3 possibilidades para o 2º lugar e 2 possibilidades para o 3º lugar → 4.3.2 = 24 possibilidades. Arranjo simples: n! An , p = (n − p)! A6, 2 + A4,3 − A5, 2 4) Calcule . A9, 2 + A8,1 6! 4! 5! + − A6, 2 + A4,3 − A5, 2 (6 − 2)! (4 − 3)! (5 − 2)! 30 + 24 − 20 34 17 = = = = A9, 2 + A8,1 9! 8! 72 + 8 80 40 + (9 − 2)! (8 − 1)! 1
  • 2. 5) Quantos números de 3 algarismos distintos podemos formar com o algarismos do sistema decimal (0,1,2,3,4,5,6,7,8,9) sem os repetir, de modo que : a) COMECEM COM 1. R : O número pode possuir três algarismos, sendo que para o primeiro existe apenas 1 possibilidade (1) e para os outros dois ainda existem 9 números disponíveis : 9! 9! 9.8.7! 1. A9, 2 = = = = 9.8 = 72 números. (9 − 2)! 7! 7! b) COMECEM COM 2 E TERMINEM COM 5. R : Para o primeiro algarismo existe apenas 1 possibilidade (2), e para o terceiro também existe apenas 1 possibilidade (5). Para o segundo ainda existem 8 possibilidades : 8! 8! 8.7! 1.1. A8,1 = = = = 8 números. (8 − 1)! 7! 7! c) SEJAM DIVISÍVEIS POR 5. R : Para um número ser divisível 5, ele deve terminar com 0 ou com 5. Primeiramente vamos calcular o número de divisíveis por 5 que terminam com 0 : → Para o terceiro algarismo existe apenas 1 possibilidade (0), e para os dois primeiros ainda existem 9 números disponíveis. Portanto o número de divisíveis por 5 que terminam com 0 é : 9! 9! 9.8.7! 1. A9, 2 = = = = 9.8 = 72 números. (9 − 2)! 7! 7! → Agora calculamos quantos divisíveis por 5 terminam com 5 : para o terceiro algarismo existe apenas uma possibilidade (5). Para o primeiro algarismo existem ainda 8 possibilidades, pois o número não pode começar com 0 (senão seria um número de 2 algarismos). E para o segundo algarismo também existem 8 possibilidades (o segundo algarismo pode ser 0). 8! 8! 8! 8! 8.7! 8.7! 1. A8,1 . A8,1 = . = . = . = 8.8 = 64 números. (8 − 1)! (8 − 1)! 7! 7! 7! 7! Resposta : O número de divisíveis por 5 é 72 + 64 = 136 números. 6) Quantos são os números compreendidos entre 2000 e 3000 formados por algarismos distintos escolhidos entre 1,2,3,4,5,6,7,8 e 9? R : O número deve ter quatro algarismos (pois está entre 2000 e 3000). Para o primeiro algarismo existe apenas uma possibilidade (2), e para os outros três ainda existem 8 números disponíveis, então : 8! 8! 8.7.6.5! 1. A8,3 = = = = 8.7.6 = 336 números. (8 − 3)! 5! 5! 2
  • 3. Permutação Simples: É um caso particular de arranjo simples. É o tipo de agrupamento ordenado onde entram todos os elementos. Pn = n! 7) Quantos números de 5 algarismos distintos podem ser formados por 1,2,3,5 e 8? P5 = 5! = 5.4.3.2.1 = 120 números. 8) Quantos anagramas da palavra EDITORA : a) COMEÇAM POR A. Para a primeira letra existe apenas uma possibilidade (A), e para as outras 6 letras existem 6 possibilidades. Então o total é : 1.P6 = 1.6!= 6.5.4.3.2.1 = 720 anagramas. b) COMEÇAM POR A e terminam com E. Para a primeira letra existe 1 possibilidade (A), e para última também só existe 1 (E), e para as outras 5 letras existem 5 possibilidades. Então o total é : 1.1.P5 = 1.1.5!= 5.4.3.2.1 = 120 anagramas. 8) Calcule de quantas maneiras podem ser dipostas 4 damas e 4 cavalheiros, numa fila, de forma que não fiquem juntos dois cavalheiros e duas damas. R :Existem duas maneiras de fazer isso : C - D - C - D - C - D - C - D ou D - C - D - C - D - C - D - C Colocando um cavalheiro na primeira posição temos como número total de maneiras : P4 .P4 = 4!.4!= 24.24 = 576 maneiras. Colocando uma dama na primeira posição temos também : P4 .P4 = 4!.4!= 24.24 = 576 maneiras. Portanto o total é 576 + 576 = 1152 maneiras. Combinação Simples: é o tipo de agrupamento em que um grupo difere do outro apenas pela natureza dos elementos componentes. n! Cn, p = p!(n − p )! 3
  • 4. 9) Resolver a equação C m,3 − C m , 2 = 0. m! m! − =0 3!(m − 3)! 2!(m − 2)! m.(m − 1).(m − 2).(m − 3)! m.(m − 1).(m − 2)! − =0 3!( m − 3)! 2!(m − 2)! m.(m − 1).(m − 2) m.(m − 1) − =0 3! 2! m 3 − 2m 2 − m 2 + 2m m 2 − m − =0 6 2 m 3 − 3m 2 + 2m − 3m 2 + 3m = 0 ⇒ m 3 − 6m 2 + 5m = 0 6 6 ± 16 m ' = 5 m 2 − 6m + 5 = 0 ⇒ m = ⇒  2 m ' ' = 1 Resposta : m = 5. obs : m = 1 não é a resposta porque não pode haver C1,3 . 10) Com 10 espécies de frutas, quantos tipos de salada, contendo 6 espécies diferentes podem ser feitas? 10! 10.9.8.7.6! 5040 5040 C10,6 = = = = = 210 tipos de saladas. 6!.(10 − 6)! 6!.4! 4! 24 11) Numa reunião com 7 rapazes e 6 moças, quantas comissões podemos formar com 3 rapazes e 4 moças? RAPAZES - C 7 ,3 MOÇAS - C 6, 4 O resultado é o produto C 7 ,3 .C 6, 4 . 7! 6! 7.6.5.4! 6.5.4! 210 30 . = . = . = 35.15 = 525 comissões. 3!(7 − 3)! 4!(6 − 4)! 3!.4! 4!.2! 3! 2 TEORIA DOS CONJUNTOS 4
  • 5. Símbolos : pertence : existe : não pertence : não existe : está contido : para todo (ou qualquer que seja) : não está contido : conjunto vazio : contém N: conjunto dos números naturais : não contém Z : conjunto dos números inteiros / : tal que Q: conjunto dos números racionais : implica que Q'= I: conjunto dos números irracionais : se, e somente se R: conjunto dos números reais Veja também: Símbolos das operações - Conceitos sobre conjuntos TABELA TRIGONOMÉTRICA 5
  • 6. Ângulo sen cos tg Ângulo sen cos tg 1 0,017452 0,999848 0,017455 46 0,71934 0,694658 1,03553 2 0,034899 0,999391 0,034921 47 0,731354 0,681998 1,072369 3 0,052336 0,99863 0,052408 48 0,743145 0,669131 1,110613 4 0,069756 0,997564 0,069927 49 0,75471 0,656059 1,150368 5 0,087156 0,996195 0,087489 50 0,766044 0,642788 1,191754 6 0,104528 0,994522 0,105104 51 0,777146 0,62932 1,234897 7 0,121869 0,992546 0,122785 52 0,788011 0,615661 1,279942 8 0,139173 0,990268 0,140541 53 0,798636 0,601815 1,327045 9 0,156434 0,987688 0,158384 54 0,809017 0,587785 1,376382 10 0,173648 0,984808 0,176327 55 0,819152 0,573576 1,428148 11 0,190809 0,981627 0,19438 56 0,829038 0,559193 1,482561 12 0,207912 0,978148 0,212557 57 0,838671 0,544639 1,539865 13 0,224951 0,97437 0,230868 58 0,848048 0,529919 1,600335 14 0,241922 0,970296 0,249328 59 0,857167 0,515038 1,664279 15 0,258819 0,965926 0,267949 60 0,866025 0,5 1,732051 16 0,275637 0,961262 0,286745 61 0,87462 0,48481 1,804048 17 0,292372 0,956305 0,305731 62 0,882948 0,469472 1,880726 18 0,309017 0,951057 0,32492 63 0,891007 0,45399 1,962611 19 0,325568 0,945519 0,344328 64 0,898794 0,438371 2,050304 20 0,34202 0,939693 0,36397 65 0,906308 0,422618 2,144507 21 0,358368 0,93358 0,383864 66 0,913545 0,406737 2,246037 22 0,374607 0,927184 0,404026 67 0,920505 0,390731 2,355852 23 0,390731 0,920505 0,424475 68 0,927184 0,374607 2,475087 24 0,406737 0,913545 0,445229 69 0,93358 0,358368 2,605089 25 0,422618 0,906308 0,466308 70 0,939693 0,34202 2,747477 26 0,438371 0,898794 0,487733 71 0,945519 0,325568 2,904211 27 0,45399 0,891007 0,509525 72 0,951057 0,309017 3,077684 28 0,469472 0,882948 0,531709 73 0,956305 0,292372 3,270853 29 0,48481 0,87462 0,554309 74 0,961262 0,275637 3,487414 30 0,5 0,866025 0,57735 75 0,965926 0,258819 3,732051 31 0,515038 0,857167 0,600861 76 0,970296 0,241922 4,010781 32 0,529919 0,848048 0,624869 77 0,97437 0,224951 4,331476 33 0,544639 0,838671 0,649408 78 0,978148 0,207912 4,70463 34 0,559193 0,829038 0,674509 79 0,981627 0,190809 5,144554 35 0,573576 0,819152 0,700208 80 0,984808 0,173648 5,671282 36 0,587785 0,809017 0,726543 81 0,987688 0,156434 6,313752 37 0,601815 0,798636 0,753554 82 0,990268 0,139173 7,11537 38 0,615661 0,788011 0,781286 83 0,992546 0,121869 8,144346 39 0,62932 0,777146 0,809784 84 0,994522 0,104528 9,514364 40 0,642788 0,766044 0,8391 85 0,996195 0,087156 11,43005 41 0,656059 0,75471 0,869287 86 0,997564 0,069756 14,30067 42 0,669131 0,743145 0,900404 87 0,99863 0,052336 19,08114 6
  • 7. Vetores Reta Orientada - Eixo Uma reta r é orientada quando fixa nela um sentido de percurso, considerado positivo e indicado por uma seta. Segmento orientado Um segmento orientado é determinado por um par ordenado de pontos, o primeiro chamado origem do segmento, o segundo chamado extremidade. Segmento Nulo Um segmento nulo é aquele cuja extremidade coincide com a origem. Segmentos Opostos Se AB é um segmento orientado, o segmento orientado BA é oposto de AB. Medida de um Segmento Fixada uma unidade de comprimento, cada segmento orientado pode-se associar um número real, não negativo, que é a medida do segmento em relação aquela unidade. A medida do segmento orientado é o seu comprimento ou seu módulo. O comprimento do segmento AB é indicado por . Assim, o comprimento do segmento AB representado na figura abaixo é de 5 unidades de comprimento: = 5 u.c. Observações a. Os segmentos nulos têm comprimento igual a zero b. = . Vetores Direção e Sentido Dois segmentos orientados não nulos AB e CD têm a mesma direção se as retas suportes desses segmentos são paralelas: 7
  • 8. ou coincidentes Observações a. Só se pode comparar os sentidos de dois segmentos orientados se eles têm mesma direção. b. Dois Segmentos orientados opostos têm sentidos contrários. Segmentos Equipolentes Dois segmentos orientados AB e CD são equipolentes quando têm a mesma direção, o mesmo sentido e o mesmo comprimento. Se os segmentos orientados AB e CD não pertencem à mesma reta. Na segunda figura abaixo, para que AB seja equipolente a CD é necessário que AB//CD e AC/BD, isto é, ABCD deve ser um paralelogramo. Observações a. Dois segmentos nulos são sempre equipolentes. b. A equipolência dos segmentos AB e CD é representada por AB ~ CD. 8
  • 9. Propriedades da Equipolência I. AB ~ AB (reflexiva). II. Se AB ~ CD, CD ~ AB (simétrica). III. Se AB ~ CD e CD ~ EF, AB ~ EF (transitiva). IV. Dado o segmento orientado AB e um ponto C, existe um único ponto D tal que AB ~ CD. Vetor Vetor determinado por um segmento orientado AB é o conjunto de todos os segmentos orientados equipolentes a AB. Se indicarmos com este conjunto, simbolicamente poderemos escrever: = {XY/XY ~ AB} onde XY é um segmento qualquer do conjunto. O vetor determinado por AB é indicado por ou B - A ou . um mesmo vetor é determinado por uma infinidade de segmentos orientados, chamados representantes desse vetor, e todos equipolentes entre si. Assim, um segmento determina um conjunto que é o vetor, e qualquer um destes representantes determina o mesmo vetor. Usando um pouco mais nossa capacidade de abstração, se considerarmos todos os infinitos segmentos orientados de origem comum, estaremos caracterizando, através de representantes, a totalidade dos vetores do espaço. Ora, cada um destes segmentos é um representante de um só vetor. Conseqüentemente, todos os vetores se acham representados naquele conjunto que imaginamos. As características de um vetor são as mesmas de qualquer um de seus representantes, isto é: o módulo, a direção e o sentido do vetor são o módulo, direção e o sentido de qualquer um de seus representantes. O módulo de se indica por | | . Vetores iguais Dois vetores e são iguais se, e somente se, AB ~ CD. Vetor Nulo Os segmentos nulos, por serem equipolentes entre si, determinam um único vetor, chamado vetor nulo ou vetor zero, e que é indicado por . Vetores Opostos Dado um vetor = , o vetor é o oposto de e se indica por ou por . 9
  • 10. Vetor Unitário Um vetor é unitário se | | = 1. Versor Versor de um vetor não nulo é o vetor unitário de mesma direção e mesmo sentido de . Por exemplo, tomemos um vetor de módulo 3. Os vetores e da figura são vetores unitários, pois ambos têm módulo 1. No entanto, apenas tem a mesma direção e o mesmo sentido de . Portanto, este é o versor de . Vetores Colineares Dois vetores e são colineares se tiverem a mesma direção. Em outras palavras: e são colineares se tiverem representantes AB e CD pertencentes a uma mesma reta ou a retas paralelas. Vetores Coplanares 10
  • 11. Se os vetores não nulos , e (não importa o número de vetores) possuem representantes AB, CD e EF pertencentes a um mesmo plano π, diz-se que eles são coplanares. Dois vetores e quaisquer são são sempre coplanares, pois podemos sempre tomar um ponto no espaço e, com origem nele, imaginar os dois representantes de e pertencendo a um plano p que passa por este ponto. Três vetores poderão ou não ser coplanares. , e são coplanares , e não são coplanares Soma de vetores Se v=(a,b) e w=(c,d), definimos a soma de v e w, por: v + w = (a+c,b+d) Propriedades da soma de vetores I) Comutativa: Para todos os vetores u e v de R2: 11
  • 12. v+w=w+v II) Associativa: Para todos os vetores u, v e w de R2: u + (v + w) = (u + v) + w III) Elemento neutro: Existe um vetor O=(0,0) em R2 tal que para todo vetor u de R2, se tem: O+u=u IV) Elemento oposto: Para cada vetor v de R2, existe um vetor -v em R2 tal que: v + (-v) = O Diferença de vetores Se v=(a,b) e w=(c,d), definimos a diferença entre v e w, por: v - w = (a-c,b-d) Produto de um escalar por um vetor Se v=(a,b) é um vetor e c é um número real, definimos a multiplicação de c por v, como: c.v = (ca,cb) Propriedades do produto de escalar por vetor Quaisquer que sejam k e c escalares, v e w vetores: • 1v=v • (k c) v = k (c v) = c (k v) • k v = c v implica k = c, se v for não nulo • k (v+w) = k v + k w • (k + c)v = k v + c v Módulo de um vetor O módulo ou comprimento do vetor v=(a,b) é um número real não negativo, definido por: Vetor unitário Vetor unitário é o que tem o módulo igual a 1. Existem dois vetores unitários que formam a base canônica para o espaço R2, que são dados por: i = (1,0) j = (0,1) Para construir um vetor unitário u que tenha a mesma direção e sentido que um outro vetor v, basta dividir o vetor v pelo seu módulo, isto é: Observação: 12
  • 13. Para construir um vetor u paralelo a um vetor v, basta tomar u=cv onde c é um escalar não nulo. Nesse caso, u e v serão paralelos. Se c = 0 então u será o vetor nulo. Se 0 < c < 1 então u terá comprimento menor do que v. Se c > 1 então u terá comprimento maior do que v. Se c < 0 então u terá sentido oposto ao de v. Próximo tópico: Produto escalar, Propriedades do produto escalar, Ângulos entre dois vetores, Vetores ortogonais PROBABILIDADE A história da teoria das probabilidades, teve início com os jogos de cartas, dados e de roleta. Esse é o motivo da grande existência de exemplos de jogos de azar no estudo da probabilidade. A teoria da probabilidade permite que se calcule a chance de ocorrência de um número em um experimento aleatório. Experimento Aleatório 13
  • 14. É aquele experimento que quando repetido em iguais condições, podem fornecer resultados diferentes, ou seja, são resultados explicados ao acaso. Quando se fala de tempo e possibilidades de ganho na loteria, a abordagem envolve cálculo de experimento aleatório. Espaço Amostral É o conjunto de todos os resultados possíveis de um experimento aleatório. A letra que representa o espaço amostral, é S. Exemplo: Lançando uma moeda e um dado, simultaneamente, sendo S o espaço amostral, constituído pelos 12 elementos: S = {K1, K2, K3, K4, K5, K6, R1, R2, R3, R4, R5, R6} 1. Escreva explicitamente os seguintes eventos: A={caras e m número par aparece}, B={um número primo aparece}, C={coroas e um número ímpar aparecem}. 2. Idem, o evento em que: a) A ou B ocorrem; b) B e C ocorrem; c) Somente B ocorre. 3. Quais dos eventos A,B e C são mutuamente exclusivos Resolução: 1. Para obter A, escolhemos os elementos de S constituídos de um K e um número par: A={K2, K4, K6}; Para obter B, escolhemos os pontos de S constituídos de números primos: B={K2,K3,K5,R2,R3,R5} Para obter C, escolhemos os pontos de S constituídos de um R e um número ímpar: C={R1,R3,R5}. 2. (a) A ou B = AUB = {K2,K4,K6,K3,K5,R2,R3,R5} (b) B e C = B ∩ C = {R3,R5} (c) Escolhemos os elementos de B que não estão em A ou C; B ∩ Ac ∩ Cc = {K3,K5,R2} 3. A e C são mutuamente exclusivos, porque A ∩ C = ∅ Conceito de probabilidade Se em um fenômeno aleatório as possibilidades são igualmente prováveis, então a probabilidade de ocorrer um evento A é: 14
  • 15. Por, exemplo, no lançamento de um dado, um número par pode ocorrer de 3 maneiras diferentes dentre 6 igualmente prováveis, portanto, P = 3/6= 1/2 = 50% Dizemos que um espaço amostral S (finito) é equiprovável quando seus eventos elementares têm probabilidades iguais de ocorrência. Num espaço amostral equiprovável S (finito), a probabilidade de ocorrência de um evento A é sempre: Propriedades Importantes: 1. Se A e A’ são eventos complementares, então: P( A ) + P( A' ) = 1 2. A probabilidade de um evento é sempre um número entre ∅ (probabilidade de evento impossível) e 1 (probabilidade do evento certo). Probabilidade Condicional Antes da realização de um experimento, é necessário que já tenha alguma informação sobre o evento que se deseja observar. Nesse caso, o espaço amostral se modifica e o evento tem a sua probabilidade de ocorrência alterada. Fórmula de Probabilidade Condicional P(E1 e E2 e E3 e ...e En-1 e En) é igual a P(E1).P(E2/E1).P(E3/E1 e E2)...P(En/E1 e E2 e ...En-1). Onde P(E2/E1) é a probabilidade de ocorrer E2, condicionada pelo fato de já ter ocorrido E1; P(E3/E1 e E2) é a probabilidade ocorrer E3, condicionada pelo fato de já terem ocorrido E1 e E2; P(Pn/E1 e E2 e ...En-1) é a probabilidade de ocorrer En, condicionada ao fato de já ter ocorrido E1 e E2...En-1. Exemplo: Uma urna tem 30 bolas, sendo 10 vermelhas e 20 azuis. Se ocorrer um sorteio de 2 bolas, uma de cada vez e sem reposição, qual será a probabilidade de a primeira ser vermelha e a segunda ser azul? Resolução: Seja o espaço amostral S=30 bolas, e considerarmos os seguintes eventos: A: vermelha na primeira retirada e P(A) = 10/30 B: azul na segunda retirada e P(B) = 20/29 Assim: P(A e B) = P(A).(B/A) = 10/30.20/29 = 20/87 Eventos independentes Dizemos que E1 e E2 e ...En-1, En são eventos independentes quando a probabilidade de ocorrer um deles não depende do fato de os outros terem ou não terem ocorrido. 15
  • 16. Fórmula da probabilidade dos eventos independentes: P(E1 e E2 e E3 e ...e En-1 e En) = P(E1).P(E2).p(E3)...P(En) Exemplo: Uma urna tem 30 bolas, sendo 10 vermelhas e 20 azuis. Se sortearmos 2 bolas, 1 de cada vez e repondo a sorteada na urna, qual será a probabilidade de a primeira ser vermelha e a segunda ser azul? Resolução: Como os eventos são independentes, a probabilidade de sair vermelha na primeira retirada e azul na segunda retirada é igual ao produto das probabilidades de cada condição, ou seja, P(A e B) = P(A).P(B). Ora, a probabilidade de sair vermelha na primeira retirada é 10/30 e a de sair azul na segunda retirada 20/30. Daí, usando a regra do produto, temos: 10/30.20/30=2/9. Observe que na segunda retirada forma consideradas todas as bolas, pois houve reposição. Assim, P(B/A) =P(B), porque o fato de sair bola vermelha na primeira retirada não influenciou a segunda retirada, já que ela foi reposta na urna. Probabilidade de ocorrer a união de eventos Fórmula da probabilidade de ocorrer a união de eventos: P(E1 ou E2) = P(E1) + P(E2) - P(E1 e E2) De fato, se existirem elementos comuns a E1 e E2, estes eventos estarão computados no cálculo de P(E1) e P(E2). Para que sejam considerados uma vez só, subtraímos P(E1 e E2). Fórmula de probabilidade de ocorrer a união de eventos mutuamente exclusivos: P(E1 ou E2 ou E3 ou ... ou En) = P(E1) + P(E2) + ... + P(En) Exemplo: Se dois dados, azul e branco, forem lançados, qual a probabilidade de sair 5 no azul e 3 no branco? Considerando os eventos: A: Tirar 5 no dado azul e P(A) = 1/6 B: Tirar 3 no dado branco e P(B) = 1/6 Sendo S o espaço amostral de todos os possíveis resultados, temos: n(S) = 6.6 = 36 possibilidades. Daí, temos:P(A ou B) = 1/6 + 1/6 – 1/36 = 11/36 Exemplo: Se retirarmos aleatoriamente uma carta de baralho com 52 cartas, qual a probabilidade de ser um 8 ou um Rei? Sendo S o espaço amostral de todos os resultados possíveis, temos: n(S) = 52 cartas. Considere os eventos: A: sair 8 e P(A) = 4/52 B: sair um rei e P(B) = 4/52 Assim, P(A ou B) = 4/52 + 4/52 – 0 = 8/52 = 2/13. Note que P(A e B) = 0, pois uma carta não pode ser 8 e rei ao mesmo tempo. Quando isso ocorre dizemos que os eventos A e B são mutuamente exclusivos. DEFINIÇÃO DE LOGARITMO 16
  • 17. a x = b ⇔ x = log a b sendo b>0 ,a>0 e a≠1 Na igualdade x = log a b obtemos : a= base do logaritmo b= logaritmando ou antilogaritmo x= logaritmo Exemplos : 1) log 2 32 = 5 pois 2 5 = 32 2) log 4 16 = 2 pois 4 2 = 16 3) log 5 1 = 0 pois 5 0 = 1 Consequências da definição Sendo b>0 ,a>0 e a≠1 e m um número real qualquer, temos a seguir algumas consequências da definição de logaritmo: log a 1 = 0 log a a = 1 log a a m = m a log a b = b log a b = log a c ⇔ b = c Propriedades operatórias dos logaritmos 1) Logaritmo do produto: log a ( x. y ) = log a x + log a y (a>0, a≠1, x>0 e y>0) 2) Logaritmo do quociente:  x (a>0, a≠1, x>0 e y>0) log a   = log a x − log a y  y   3) Logaritmo da potência: log a x m = m. log a x (a>0, a≠1, x>0 e m ∈ℜ) m n x m =x n Caso particular: como , temos: m m log a x = log a x = n m n . log a x n Cologaritmo 17
  • 18. Chamamos de cologaritmo de um número positivo b numa base a (a>0, a≠1) e indicamos cologa b o logaritmo inverso desse número b na base a 1 colog a b = log a (a>0, a≠1 e b>0) b 1 Como log a = log a 1 − log a b = 0 − log a b = − log a b, podemos também escrever : b colog a b = − log a b Mudança de base Em algumas situações podemos encontrar no cálculo vários logaritmos em bases diferentes. Como as propriedades logarítmicas só valem para logaritmos numa mesma base, é necessário fazer, antes, a conversão dos logaritmos de bases diferentes para uma única base conveniente. Essa conversão chama-se mudança de base. Para fazer a mudança de uma base a para uma outra base b usa-se: log b x log a x = log b a MATRIZES E DETERMINANTES 18
  • 19. 1) Dadas as matrizes :  5 2  2 − 2 a b  A=  , B = 0 1  e X =  c d  tais que 2 A − X = B, calcule o determinante de X . − 1 1     Primeiramente encontramos a matriz X : 5 2   a b  2 − 2  2 − = − 1 1   c d  0 1        10 4  a b   2 − 2 − 2 − =  2  c d  0 1       10 − a = 2 → a = 8 4 − b = −2 → b = 6  10 − a 4 − b  2 − 2   8 6  − 2 − c 2 − d  = 0 1  ⇒  ⇒ X =      − 2 − c = 0 → c = −2 − 2 1  2 − d = 1 → d = 1  8 6 det X = = 8.1 − 6.(−2) = 8 + 12 = 20 −2 1 2 1 3 2) Encontre a solução da equação 4 − 1 n − 1 = 12. n 0 n Para achar o determinante de uma matriz 3x3 podemos utilizar a regra de Sarrus, que consiste em copiar as duas primeiras colunas à direita da matriz, e subtrair a soma dos produtos da primeira diagonal, pela soma dos produtos da segunda : 2 1 3 2 1 4 − 1 n − 1 4 − 1 = 12 ⇒ (−2n + n(n − 1) + 0) − ( −3n + 0 + 4n) = 12 n 0 n n 0 (−2n + n 2 − n) − n = 12 ⇒ n 2 − 4n − 12 = 0 4 ± 16-4.1.(-12 ) 4 ± 64 4±8 n = 6 n= ⇒ n= ⇒ n= ⇒ 2 2 2  n = −2  1 0 5 − 3 3) Sendo A = − 2 3 e B =     calcule AB.  0 4 1 2    Essa é uma questão de multiplicação de matrizes, onde estamos multiplicando uma matriz 3x2 por uma 2x2. O resultado será obtido pelo produto de cada linha da matriz A por cada coluna da matriz B. O resultado será uma matriz 3x2.  1.5 + 0.1 1.(−3) + 0.2   5 − 3 AB = (−2).5 + 3.1 (−2)(−3) + 3.2 ⇒ AB = − 7 12       0.5 + 4.1  0(−3) + 4.2  4  8 19
  • 20. 4 5  4) Sendo A =  , determine a matriz inversa da matriz A. 3 4 Sabemos que uma matriz multiplicada pela sua inversa resulta na matriz identidade, ou seja : A. A −1 = I 4a + 5c = 1 4a + 5c = 1 a = 4 4b + 5d = 0  →  4 5 a b  1 0  3a + 4c = 0 c = −3 3 4. c d  = 0 1 ⇒ 3a + 4c = 0 ⇒ 4b + 5d = 0 b = −5       →   3b + 4d = 1  3b + 4d = 1 d = 4  4 − 5 Portanto, a matriz inversa de A é A −1 =   − 3 4  O CONJUNTO DOS NÚMEROS COMPLEXOS De números complexos você deve saber : i 2 = −1 Conjugado de um número complexo : z = a + bi ⇔ z = a − bi z1 z1 .z 2 Divisão de dois números complexos : = z 2 z 2 .z 2 Módulo de um número complexo : z = a 2 + b 2 a b Argumento de um número complexo : cos(θ ) = e sen(θ ) = z z Forma trigonométrica ou polar : z = z .(cos(θ ) + i. sen(θ )) Multiplicação na forma trigonométrica : z1 .z 2 = z1 . z 2 .(cos(θ 1 + θ 2 ) + i. sen(θ 1 + θ 2 )) z1 z1 Divisão na forma trigonométrica : = .(cos(θ 1 − θ 2 ) + i. sen(θ 1 − θ 2 )) z2 z2 n Potenciação na forma trigonométrica : z n = z .(cos(nθ ) + i. sen(nθ )) 20
  • 21. Exercícios resolvidos 2+i 1) Calcule . 5 − 3i Multiplicam - se ambos os termos da fração pelo número complexo conjugado do denominador : (2 + i ) (5 + 3i ) 10 + 6i + 5i + 3i 2 10 + 11i − 3 7 + 11i 7 11 . = = = = + i (5 − 3i ) (5 + 3i ) 25 − 9i 2 25 − ( −9) 34 34 34 1− i i 2) Coloque na forma a + bi a expressão + . 1+ i i − 2 Em cada fração, multiplicamos seus termos pelo número complexo conjugado do denominador : (1 − i ) (1 − i ) i ( −2 − i ) 1 − 2i + i 2 − 2i − i 2 1 − 2i − 1 − 2i − (−1) . + . = + = + = (1 + i ) (1 − i ) (−2 + i ) (−2 − i ) 1− i 2 4−i 2 1 − (−1) 4 − (−1) − 2i 1 − 2i 1 − 2i − 5i + 1 − 2i 1 − 7i 1 7 = + = −i+ = = = − i 2 5 5 5 5 5 5 21
  • 22. 3) Calcule : 92 4 45 4 a) i 92 → 92 23 → i =1 0 b) i 45 → 44 11 → i1 = i 0 1 310 4 1081 4 c) i 310 → 308 77 → i = −1 2 d) i 1081 → 1080 270 → i 1 = i 2 1 e) i 4 n = i 4 = i 2 .i 2 = (−1).(−1) = 1 f) i 4 n +1 = i 4 n .i = 1.i = i g) i 4 n + 2 = i 4 n .i 2 = 1.(−1) = −1 h) i 4 n + 3 = i 4 n .i 3 = 1.(−i ) = −i (4 − 3i )(12 − 5i ) 4) Ache o módulo do número complexo . 2i Primeiramente colocamos o número na forma a + bi : (4 − 3i )(12 − 5i ) (− 2i ) (48 − 20i − 36i + 15i 2 ).(− 2i ) (33 − 56i ).(− 2i ) . = = = ( 2i ) (− 2i ) − 2i 2 − 2(−1) − 33 2i − 56 2 33 2 = = − 28 2 − i 2 2 Agora encontramos o módulo desse número complexo : 2  33 2  2178 8450 4225 z = a + b = (−28 2 ) +  − 2  2  = 1568 + = 2 = =  2  4 4 2 65 2 65 2 65 2 = . = → z = 2 2 2 2 5) Obtenha o argumento dos números complexos a seguir : a) z = 2 + 2 3i → z = 2 2 + (2 3 ) 2 = 4 + 12 = 16 = 4 a 2 1 cos(θ ) =  = = z 4 2 π  θ = 60 = 0 b 2 3 3  3 sen(θ ) = = = z 4 2  22
  • 23. b) z = 4i → z = 0 2 + 4 2 = 16 = 4 a 0  cos(θ ) = = =0  z 4  π  θ = 90 = 0 b 4 2 sen(θ ) = = = 1  z 4   6) Passe o número complexo z = 8i para a forma trigonométrica. z = 0 2 + 8 2 = 64 = 8 a 0  cos(θ ) = = =0  z 8  π  θ = b 8 2 sen(θ ) = = = 1  z 8   Passando para a forma trigonométrica : z = z .(cos(θ ) + i. sen(θ ))  π   π  z = 8. cos  + i. sen       2  2   π   π  7) Dados z1 = 5(cos(π ) + i. sen(π )) e z 2 = 3. cos  + i. sen  , obtenha z1 .z 2 .    3  3  z1 = (5 cos(π )) 2 + (5 sen(π )) 2 = (−5) 2 + 0 2 = 25 = 5 2 2   π    π  9 27 36 z 2 =  3 cos   +  3 sen    =     + = = 9 =3   3    3  4 4 4 a 3/ 2 1  a −5  cos(θ 2 ) = = =  cos(θ 1 ) = = = −1 z2 3 2 z1 5    π  θ1 = π 3 3  θ2 = b 0 3 sen(θ 1 ) = = =0  b  2 = 3  z1 5   sen(θ 2 ) = = z2 3 2   z1 .z 2 = z1 . z 2 .(cos(θ 1 + θ 2 ) + i. sen(θ 1 + θ 2 ))   π  π  z1 .z 2 = 5.3. cos π +  + i. sen  π +      3  3     4π   4π  z1 .z 2 = 15. cos   + i. sen      3   3  23
  • 24. POLINÔMIOS • Definição Uma função polinomial ou simplesmente polinômio, é toda função definida pela relação P(x)=anxn + an-1.xn-1 + an-2.xn-2 + ... + a2x2 + a1x + a0. Onde: an, an-1, an-2, ..., a2, a1, a0 são números reais chamados coeficientes. n ∈ IN x ∈ C (nos complexos) é a variável. GRAU DE UM POLINÔMIO: Grau de um polinômio é o expoente máximo que ele possui. Se o coeficiente an≠0, então o expoente máximo n é dito grau do polinômio e indicamos gr(P)=n. Exemplos: a) P(x)=5 ou P(x)=5.x0 é um polinômio constante, ou seja, gr(P)=0. b) P(x)=3x+5 é um polinômio do 1º grau, isto é, gr(P)=1. c) P(x)=4x5+7x4 é um polinômio do 5º grau, ou seja, gr(P)=5. Obs: Se P(x)=0, não se define o grau do polinômio. • Valor numérico O valor numérico de um polinômio P(x) para x=a, é o número que se obtém substituindo x por a e efetuando todas as operações indicadas pela relação que define o polinômio. Exemplo: Se P(x)=x3+2x2+x-4, o valor numérico de P(x), para x=2, é: P(x)= x3+2x2+x-4 P(2)= 23+2.22+2-4 P(2)= 14 Observação: Se P(a)=0, o número a chamado raiz ou zero de P(x). Por exemplo, no polinômio P(x)=x2-3x+2 temos P(1)=0; logo, 1 é raiz ou zero desse polinômio. 24
  • 25. Alguns exercícios resolvidos: 1º) Sabendo-se que –3 é raiz de P(x)=x3+4x2-ax+1, calcular o valor de a. Resolução: Se –3 é raiz de P(x), então P(-3)=0. P(-3)=0 => (-3)3+4(-3)2-a.(-3)+1 = 0 3a = -10 => a=-10/3 Resposta: a=-10/3 2º) Calcular m ∈ IR para que o polinômio P(x)=(m2-1)x3+(m+1)x2-x+4 seja: a) do 3ºgrau b) do 2º grau c) do 1º grau Resposta: a) para o polinômio ser do 3º grau, os coeficientes de x2 e x3 devem ser diferentes de zero. Então: m2-1≠0 => m2≠1 => m≠1 m+1≠0 => m≠-1 Portanto, o polinômio é do 3º grau se m≠1 e m≠-1. b) para o polinômio ser do 2º grau, o coeficiente de x3 deve ser igual a zero e o coeficiente de x2 diferente de zero. Então: m2-1=0 => m2=1 => m=±1 m+1≠0 => m≠-1 Portanto, o polinômio é do 2º grau se m=1. c) para o polinômio ser do 1º grau, os coeficientes de x2 e x3 devem ser iguais a zero. Então: m2-1=0 => m2=1 => m=±1 m+1=0 => m=-1 Portanto, o polinômio é do 1º grau se m=-1. 25
  • 26. 3º) Num polinômio P(x), do 3º grau, o coeficiente de x3 é 1. Se P(1)=P(2)=0 e P(3)=30, calcule o valor de P(-1). Resolução: Temos o polinômio: P(x)=x3+ax2+bx+c. Precisamos encontrar os valores de a,b e c (coeficientes). Vamos utilizar os dados fornecidos pelo enunciado do problema: P(1)=0 => (1)3+a.(1)2+b(1)+c = 0 => 1+a+b+c=0 => a+b+c=-1 P(2)=0 => (2)3+a.(2)2+b(2)+c = 0 => 8+4a+2b+c=0 => 4a+2b+c=-8 P(3)=30 => (3)3+a.(3)2+b(3)+c = 30 => 27+9a+3b+c=30 => 9a+3b+c=3 Temos um sistema de três variáveis: a + b + c = -1  4a + 2b + c = -8 9a + 3b + c = 3  Resolvendo esse sistema encontramos as soluções: a=9, b=-34, c=24 Portanto o polinômio em questão é P(x)= x3+9x2-34x+24. O problema pede P(-1): P(-1)= (-1)3+9(-1)2-34(-1)+24 => P(-1)=-1+9+34+24 P(-1)= 66 Resposta: P(-1)= 66 26
  • 27. • Polinômios iguais Dizemos que dois polinômios A(x) e B(x) são iguais ou idênticos (e indicamos A(x)≡B(x)) quando assumem valores numéricos iguais para qualquer valor comum atribuído à variável x. A condição para que dois polinômios sejam iguais ou idênticos é que os coeficientes dos termos correspondentes sejam iguais. Exemplo: Calcular a,b e c, sabendo-se que x2-2x+1 ≡ a(x2+x+1)+(bx+c)(x+1). Resolução: Eliminando os parênteses e somando os termos semelhantes do segundo membro temos: x2-2x+1 ≡ ax2+ax+a+bx2+bx+cx+c 1x2-2x+1 ≡ (a+b)x2+(a+b+c)x+(a+c) Agora igualamos os coeficientes correspondentes: a + b = 1  a + b + c = −2 a + c = 1  Substituindo a 1ª equação na 2ª: 1+c = -2 => c=-3. Colocando esse valor de c na 3ª equação, temos: a-3=1 => a=4. Colocando esse valor de a na 1ª equação, temos: 4+b=1 => b=-3. Resposta: a=4, b=-3 e c=-3. Obs: um polinômio é dito identicamente nulo se tem todos os seus coeficientes nulos. 27
  • 28. • Divisão de polinômios Sejam dois polinômios P(x) e D(x), com D(x) não nulo. Efetuar a divisão de P por D é determinar dois polinômios Q(x) e R(x), que satisfaçam as duas condições abaixo: 1ª) Q(x).D(x) + R(x) = P(x) 2ª) gr(R) < gr(D) ou R(x)=0 P( x) D( x ) R( x) Q( x) Nessa divisão: P(x) é o dividendo. D(x) é o divisor. Q(x) é o quociente. R(x) é o resto da divisão. Obs: Quando temos R(x)=0 dizemos que a divisão é exata, ou seja, P(x) é divisível por D(x) ou D(x) é divisor de P(x). Se D(x) é divisor de P(x) ⇔ R(x)=0 Exemplo: Determinar o quociente de P(x)=x4+x3-7x2+9x-1 por D(x)=x2+3x-2. Resolução: Aplicando o método da chave, temos: x 4 + x3 − 7 x 2 + 9 x − 1 x 2 + 3x − 2 − x 4 − 3x3 + 2 x 2 x 2 − 2 x + 1 → Q( x) − 2 x3 − 5x2 + 9 x − 1 + 2 x3 + 6 x2 − 4 x x2 + 5x − 1 − x 2 − 3x + 2 2 x + 1 → R ( x) Verificamos que: x 4  -  1 ≡ (x 2 + 3x - 2) (x 2 - 2x + 1) + (2x + 1)  + x 7x + 9x 3 2 -      P(x) D(x) Q(x) R(x) 28
  • 29. • Divisão de um polinômio por um binômio da forma ax+b Vamos calcular o resto da divisão de P(x)=4x2-2x+3 por D(x)=2x-1. Utilizando o método da chave temos: 4 x2 − 2 x + 3 2x − 1 − 4 x2 + 2 x 2x 3 Logo: R(x)=3 A raiz do divisor é 2x-1=0 => x=1/2. Agora calculamos P(x) para x=1/2. P(1/2) = 4(1/4) – 2(1/2) + 3 P(1/2) = 3 Observe que R(x) = 3 = P(1/2) Portanto, mostramos que o resto da divisão de P(x) por D(x) é igual ao valor numérico de P(x) para x=1/2, isto é, a raiz do divisor. • Teorema do resto O resto da divisão de um polinômio P(x) pelo binômio ax+b é igual a P(-b/a). Note que –b/a é a raiz do divisor. Exemplo: Calcule o resto da divisão de x2+5x-1 por x+1. Resolução: Achamos a raiz do divisor: x+1=0 => x=-1 Pelo teorema do resto sabemos que o resto é igual a P(-1): P(-1)=(-1)2+5.(-1)-1 => P(-1) = -5 = R(x) Resposta: R(x) = -5. • Teorema de D’Alembert Um polinômio P(x) é divisível pelo binômio ax+b se P(-b/a)=0 Exemplo: Determinar o valor de p, para que o polinômio P(x)=2x3+5x2-px+2 seja divisível por x-2. Resolução: Se P(x) é divisível por x-2, então P(2)=0. P(2)=0 => 2.8+5.4-2p+2=0 => 16+20-2p+2=0 => p=19 Resposta: p=19. 29
  • 30. • Divisão de um polinômio pelo produto (x-a)(x-b) Vamos resolver o seguinte problema: calcular o resto da divisão do polinômio P(x) pelo produto (x-a)(x-b), sabendo-se que os restos da divisão de P(x) por (x-a) e por (x-b) são, respectivamente, r1 e r2. Temos: a é a raiz do divisor x-a, portanto P(a)=r1 (eq. 1) b é a raiz do divisor x-b, portanto P(b)=r2 (eq. 2) E para o divisor (x-a)(x-b) temos P(x)=(x-a)(x-b) Q(x) + R(x) (eq. 3) O resto da divisão de P(x) por (x-a)(x-b) é no máximo do 1º grau, pois o divisor é do 2º grau; logo: R(x)=cx+d Da eq.3 vem: P(x)=(x-a)(x-b) Q(x) + cx + d Fazendo: x=a => P(a) = c(a)+d (eq. 4) x=b => P(b) = c(b)+d (eq. 5) Das equações 1, 2, 4 e 5 temos: ca + d = r1  cb + d = r2 Resolvendo o sistema obtemos: r1 − r2 ar − ar1 c= e d= 2 , com a ≠ b a−b a−b r −r ar − ar1 Logo : R ( x) = 1 2 x + 2 , com a ≠ b a−b a−b Observações: 1ª) Se P(x) for divisível por (x-a) e por (x-b), temos: P(a)= r1 =0 P(b)= r2 =0 Portanto, P(x) é divisível pelo produto (x-a)(x-b), pois: r1 − r2 ar − ar1 R( x) = x+ 2 = 0+0 = 0 a −b a−b 30
  • 31. 2ª) Generalizando, temos: Se P(x) é divisível por n fatores distintos (x-a1), (x-a2),..., (x-an) então P(x) é divisível pelo produto (x-a1)(x-a2)...(x-an). Exemplo: Um polinômio P(x) dividido por x dá resto 6 e dividido por (x-1) dá resto 8. Qual o resto da divisão de P(x) por x(x-1)? Resolução: 0 é a raiz do divisor x, portanto P(0)=6 (eq. 1) 1 é a raiz do divisor x-1, portanto P(1)=8 (eq. 2) E para o divisor x(x-1) temos P(x)=x(x-1) Q(x) + R(x) (eq. 3) O resto da divisão de P(x) por x(x-1) é no máximo do 1º grau, pois o divisor é do 2º grau; logo: R(x)=ax+b Da eq.3 vem: P(x)=x(x-1) Q(x) + ax + b Fazendo: x=0 => P(0) = a(0)+b => P(0) = b (eq. 4) x=1 => P(1) = a(1)+b => P(1) = a+b (eq. 5) Das equações 1, 2, 4 e 5 temos: b = 6  a + b = 8 Logo, b=6 e a=2. Agora achamos o resto: R(x) = ax+b = 2x+6 Resposta: R(x) = 2x+6. 31
  • 32. • O dispositivo de Briot-Ruffini Serve para efetuar a divisão de um polinômio P(x) por um binômio da forma (ax+b). Exemplo: Determinar o quociente e o resto da divisão do polinômio P(x)=3x3-5x2+x-2 por (x-2). Resolução: RAIZ DO DIVISOR    ES DE P(x)  COEFICIENT    2 3 −5 1 −2 ↓ 3.(2) − 5 1.( 2) + 1 3.( 2) − 2 1  3  3   4  COEFICIENTES DO QUOCIENTE Q(x) RESTO Observe que o grau de Q(x) é uma unidade inferior ao de P(x), pois o divisor é de grau 1. Resposta: Q(x)=3x2+x+3 e R(x)=4. Para a resolução desse problema seguimos os seguintes passos: 1º) Colocamos a raiz do divisor e os coeficientes do dividendo ordenadamente na parte de cima da “cerquinha”. 2º) O primeiro coeficiente do dividendo é repetido abaixo. 3º) Multiplicamos a raiz do divisor por esse coeficiente repetido abaixo e somamos o produto com o 2º coeficiente do dividendo, colocando o resultado abaixo deste. 4º) Multiplicamos a raiz do divisor pelo número colocado abaixo do 2º coeficiente e somamos o produto com o 3º coeficiente, colocando o resultado abaixo deste, e assim sucessivamente. 5º) Separamos o último número formado, que é igual ao resto da divisão, e os números que ficam à esquerda deste serão os coeficientes do quociente. 32
  • 33. • Decomposição de um polinômio em fatores Vamos analisar dois casos: 1º caso: O polinômio é do 2º grau. De uma forma geral, o polinômio de 2º grau P(x)=ax2+bx+c que admite as raízes r1 e r2 pode ser decomposto em fatores do 1º grau, da seguinte forma: ax2+bx+c = a(x-r1)(x-r2) Exemplos: 1) Fatorar o polinômio P(x)=x2-4. Resolução: Fazendo x2-4=0, obtemos as raízes r1=2 e r2=-2. Logo: x2-4 = (x-2)(x+2). 2) Fatorar o polinômio P(x)=x2-7x+10. Resolução: Fazendo x2-7x+10=0, obtemos as raízes r1=5 e r2=2. Logo: x2-7x+10 = (x-5)(x-2). 2º caso: O polinômio é de grau maior ou igual a 3. Conhecendo uma das raízes de um polinômio de 3º grau, podemos decompô-lo num produto de um polinômio do 1º grau por um polinômio do 2º grau e, se este tiver raízes, podemos em seguida decompô-lo também. Exemplo: Decompor em fatores do 1º grau o polinômio 2x3-x2-x. Resolução: 2x3-x2-x = x.(2x2-x-1)  colocando x em evidência Fazendo x.(2x2-x-1) = 0 obtemos: x=0 ou 2x2-x-1=0. Uma das raízes já encontramos (x=0). As outras duas saem da equação: 2x2-x-1=0 => r1=1 e r2=-1/2. Portanto, o polinômio 2x3-x2-x, na forma fatorada é: 2.x.(x-1).(x+(1/2)). Generalizando, se o polinômio P(x)=anxn+an-1xn-1+...+a1x+a0 admite n raízes r1, r2,..., rn, podemos decompô-lo em fatores da seguinte forma: anxn+an-1xn-1+...+a1x+a0 = an(x-r1)(x-r2)...(x-rn) Observações: 1) Se duas, três ou mais raiz forem iguais, dizemos que são raízes duplas, triplas, etc. 2) Uma raiz r1 do polinômio P(x) é dita raiz dupla ou de multiplicidade 2 se P(x) é divisível por (x-r1)2 e não por (x-r1)3. 33
  • 34. PRODUTOS NOTÁVEIS É muito comum nas expressões algébrica o aparecimento de certos produtos. Para simplificar o trabalho nos cálculos será muito útil a aplicação dos produtos notáveis. Veja a tabela abaixo: Produtos notáveis Exemplos 2 2 2 2 2 (a+b) = a +2ab+b (x+3) = x +6x+9 (a-b)2 = a2-2ab+b2 (x-3)2 = x2-6x+9 (a+b)(a-b) = a2-b2 (x+3)(x-3) = x2-9 (x+a)(x+b) = x2+(a+b)x+ab (x+2)(x+3) = x2+5x+6 (a+b)3 = a3+3a2b+3ab2+b3 (x+2)3 = x3+6x2+12x+8 (a-b)3 = a3-3a2b+3ab2-b3 (x-2)3 = x3-6x2+12x-8 (a+b)(a2-ab+b2) = a3+b3 (x+2)(x2-2x+4) = x3+8 (a-b)(a2+ab+b2) = a3-b3 (x-2)(x2+2x+4) = x3-8 ALGUNS EXERCÍCIOS RESOLVIDOS: 1) Desenvolva: a) (3x+y)2 (3x+y)2 = (3x)2+2.3x.y+y2 = 9x2+6xy+y2 b) ((1/2)+x2)2 ((1/2)+x2)2 = (1/2)2+2.(1/2).x2+(x2)2 = (1/4) +x2+x4 c) ((2x/3)+4y3)2 ((2x/3)+4y3)2 = (2x/3)2-2.(2x/3).4y3+(4y3)2= (4/9)x2-(16/3)xy3+16y6 d) (2x+3y)3 (2x+3y)3 = (2x)3+3.(2x)2.3y+3.2x.(3y)2+(3y)3 = 8x3+36x2y+54xy2+27y3 e) (x4+(1/x2))3 (x4+(1/x2))3 = (x4)3+3.(x4)2.(1/x2)+3.x4.(1/x2)2+(1/x2)3 = x12+3x6+3+(1/x6) f) ((2x/3)+(4y/5)).((2x/3)-(4y/5)) ((2x/3)+(4y/5)).((2x/3)-(4y/5)) = (2x/3)2-(4y/5)2 = (4/9)x2-(16/25)y2 2) Efetue as multiplicações: a) (x-2)(x-3) (x-2)(x-3) = x2+((-2)+(-3))x+(-2).(-3) = x2-5x+6 b) (x+5)(x-4) (x+5)(x-4) = x2+(5+(-4))x+5.(-4) = x2+x-20 34
  • 35. 3) Simplifique as expressões: a) (x+y)2–x2-y2 (x+y)2–x2-y2 = x2+2xy+y2–x2-y2 = 2xy b) (x+2)(x-7)+(x-5)(x+3) (x+2)(x-7)+(x-5)(x+3) = x2+(2+(-7))x+2.(-7) + x2+(-5+3)x+3.(-5) = x2-5x-14+ x2-2x-15 = 2x2-7x-29 c) (2x-y)2-4x(x-y) (2x-y)2-4x(x-y) = (2x)2-2.2x.y+y2-4x2+4xy = 4x2-4xy+y2-4x2+4xy = y2 35
  • 36. Progressões Aritméticas Progressão aritmética é uma sequência numérica na qual, a partir do segundo, cada termo é igual à soma de seu antecessor com uma constante, denominada razão. Fórmula do termo geral de uma P.A. : a n = a1 + (n − 1).r (a1 + a n ).n Soma de termos de uma P.A. finita : S n = 2 Logo abaixo temos alguns exercícios de progressões aritméticas resolvidos. 1) Dada a P.A. (-19,-15,-11,...) calcule o seu enésimo termo. Primeiramente encontramos a razão : r = a2 − a1 ⇒ r = −15 − (−19) ⇒ r = 4. Logo, o termo geral é : an = a1 + (n − 1).r ⇒ an = −19 + (n − 1).4 ⇒ an = −19 + 4n − 4 ⇒ an = 4n − 23 2) Interpole seis meios aritméticos entre –8 e 13. No problema : a1 = −8, an = 13, n = 8 (pois 6 meios aritméticos serão interpolados entre os dois extremos, que são - 8 e 13. Logo, existem 8 termos na P.A.). Para interpolar os valores, devemos encontrar a razão : an = a1 + (n − 1).r ⇒ 13 = −8 + (8 − 1).r ⇒ 13 = −8 + 7 r ⇒ 13 + 8 = 7 r ⇒ 21 7r = 21 ⇒ r = ⇒ r = 3. 7 Encontrada a razão, basta interpolar os meios aritméticos : - 8, - 5, - 2, 1, 4, 7, 10, 13 36
  • 37. 3) Escreva uma P.A. de três termos, sabendo que a soma desses termos vale 12 e que a soma de seus quadrados vale 80. a1 + a 2 + a 3 = 12   2 a1 + a 2 2 + a3 2 = 80  Sabemos que a 2 = a1 + r e que a 3 = a1 + 2r. Então substituimos no sistema acima : a1 + (a1 + r ) + (a1 + 2r ) = 12 3a1 + 3r = 12  2 ⇒  2 2 2 ⇒ a1 + (a1 + r ) + (a1 + 2r ) = 80 a1 + a1 + 2a1 r + r + a1 + 4a1 r + 4r = 80 2 2 2 2  12 − 3r 3a1 + 3r = 12 → a1 = → a1 = 4 − r ⇒  3 3a 2 + 6a r + 5r 2 = 80  1 1 Substituindo na segunda equação temos : 3(4 − r ) 2 + 6(4 − r )r + 5r 2 = 80 3(16 − 8r + r 2 ) + (24 − 6r )r + 5r 2 = 80 48 − 24r + 3r 2 + 24r − 6r 2 + 5r 2 = 80 48 + 2r 2 = 80 → 2r 2 = 80 − 48 → 2r 2 = 32 → r 2 = 16 → r = 16 → r = ±4 Agora encontramos o primeiro termo : 1) Para r = 4 : a1 = 4 - r → a 1 = 4 - 4 → a 1 = 0 P.A : (0,4,8) 1) Para r = −4 : a1 = 4 - r → a 1 = 4 - (-4) → a 1 = 8 P.A : (8,4,0) Resposta : (0,4,8) ou (8,4,0). 4) Calcule quantos números inteiros existem entre 13 e 247 que não são múltiplos de 3. Entre 13 e 247 existem 233 números. Para calcular quantos números NÃO são múltiplos de 3, nós devemos calcular primeiramente quantos números SÃO múltiplos de 3, e logo após subtrair o número total de números (233) pelo número de múltiplos, o que dará como resultado o número de NÃO múltiplos. Para calcular o número de múltiplos de 3 : a1 = 15 (pois é o primeiro múltiplo de 3 depois do 13) r = 3, a n = 246 (pois é o último múltiplo de 3 antes do 247). Basta achar o n, que é o número de múltiplos : 234 a n = a1 + (n − 1).r → 246 = 15 + (n - 1)3 → 231 = 3n - 3 → n = → n = 78 3 Dos 233 números, 78 são múltiplos de 3, logo 155 não são múltiplos de 3. 37
  • 38. 5) Encontre o valor de x para que a sequência (2x, x+1, 3x) seja uma progressão aritmética. Para ser uma P.A. : a3 − a 2 = a 2 − a1 3 x − ( x + 1) = ( x + 1) − 2 x 2x − 1 = 1 − x 2 2x + x = 1 + 1 → 3x = 2 → x= 3 6) Numa progressão aritmética em que a2+a7=a4+ak, o valor de k é: (a1 + r ) + (a1 + 6r ) = (a1 + 3r ) + a k 2a1 + 7 r = a1 + 3r + a k 2a1 − a1 + 7 r − 3r = a k → a k = a1 + 4r Logo k = 5, pois a5 = a1 + 4r. 7) Se Sn é a soma dos n primeiros termos da progressão aritmética (-90,-86,-82,...) então o menor valor de n para que se tenha Sn>0 é: r = 4  Pelo enunciado, obtemos os seguintes dados : a1 = −90 a = 94 (pois a S deve ser maior que zero)  n n Basta encontrar o número de termos : a n = a1 + (n − 1).r 94 = −90 + (n − 1).4 94 + 90 = 4n − 4 188 184 + 4 = 4n → n = → n = 47 4 8) A soma dos n primeiros números pares positivos é 132. Encontre o valor de n. r = 2 ; a1 = 2 ; S n = 132 a n = a1 + (n − 1).r → a n = 2 + (n − 1).2 → a n = 2 + 2n − 2 → a n = 2n Substituindo na fórmula da soma temos : ( a + a n ).n ( 2 + 2n) n Sn = 1 → 132 = → n 2 + n − 132 = 0 2 2 − 1 ± 1 + 4.1.132 − 1 ± 529 − 1 ± 23 n = −12 n= = = = ⇒ n = 11 2 2 2 n = 11 PROGRESSÕES GEOMÉTRICAS 38
  • 39. Podemos definir progressão geométrica, ou simplesmente P.G., como uma sucessão de números reais obtida, com exceção do primeiro, multiplicando o número anterior por uma quantidade fixa q, chamada razão. Podemos calcular a razão da progressão, caso ela não esteja suficientemente evidente, dividindo entre si dois termos consecutivos. Por exemplo, na sucessão (1, 2, 4, 8,...), q = 2. Cálculos do termo geral Numa progressão geométrica de razão q, os termos são obtidos, por definição, a partir do primeiro, da seguinte maneira: a1 a2 a3 ... a20 ... an ... a1 a1xq a1xq2 ... a1xq19 a1xqn-1 ... Assim, podemos deduzir a seguinte expressão do termo geral, também chamado enésimo termo, para qualquer progressão geométrica. an = a1 x qn-1 Portanto, se por exemplo, a1 = 2 e q = 1/2, então: an = 2 x (1/2)n-1 Se quisermos calcular o valor do termo para n = 5, substituindo-o na fórmula, obtemos: a5 = 2 x (1/2)5-1 = 2 x (1/2)4 = 1/8 A semelhança entre as progressões aritméticas e as geométricas é aparentemente grande. Porém, encontramos a primeira diferença substancial no momento de sua definição. Enquanto as progressões aritméticas formam-se somando-se uma mesma quantidade de forma repetida, nas progressões geométricas os termos são gerados pela multiplicação, também repetida, por um mesmo número. As diferenças não param aí. Observe que, quando uma progressão aritmética tem a razão positiva, isto é, r > 0, cada termo seu é maior que o anterior. Portanto, trata-se de uma progressão crescente. Ao contrário, se tivermos uma progressão aritmética com razão negativa, r < 0, seu comportamento será decrescente. Observe, também, a rapidez com que a progressão cresce ou diminui. Isto é conseqüência direta do valor absoluto da razão, |r|. Assim, quanto maior for r, em valor absoluto, maior será a velocidade de crescimento e vice-versa. Soma dos n primeiros termos de uma PG Seja a PG (a1, a2, a3, a4, ... , an , ...) . Para o cálculo da soma dos n primeiros termos Sn, Vamos considerar o que segue: Sn = a1 + a2 + a3 + a4 + ... + an-1 + an Multiplicando ambos os membros pela razão q vem: Sn.q = a1 . q + a2 .q + .... + an-1 . q + an .q Conforme a definição de PG, podemos reescrever a expressão como: Sn . q = a2 + a3 + ... + an + an . q 39
  • 40. Observe que a2 + a3 + ... + an é igual a Sn - a1 . Logo, substituindo, vem: S n . q = S n - a1 + a n . q Daí, simplificando convenientemente, chegaremos à seguinte fórmula da soma: Se substituirmos an = a1 . qn-1 , obteremos uma nova apresentação para a fórmula da soma, ou seja: Exemplo: Calcule a soma dos 10 primeiros termos da PG (1,2,4,8,...) Temos: Observe que neste caso a1 = 1. • Soma dos termos de uma PG decrescente e ilimitada Considere uma PG ILIMITADA ( infinitos termos) e decrescente. Nestas condições, podemos considerar que no limite teremos an = 0. Substituindo na fórmula anterior, encontraremos: Exemplo: Resolva a equação: x + x/2 + x/4 + x/8 + x/16 + ... =100 O primeiro membro é uma PG de primeiro termo x e razão 1/2. Logo, substituindo na fórmula, vem: Dessa equação encontramos como resposta x = 50. 40
  • 41. IDENTIDADES TRIGONOMÉTRICAS sen( x) π 1) tg ( x) = Relação válida para todo x ≠ + kπ cos( x) 2 cos( x) 2) cot g ( x) = Relação válida para todo x ≠ kπ sen( x) 1 π 3) sec( x) = Relação válida para todo x ≠ + kπ cos( x) 2 1 4) cos ec( x) = Relação válida para todo x ≠ kπ sen( x) 5) sen 2 ( x) + cos 2 ( x) = 1 Fórmulas da Adição 6) sen(a + b) = sen(a). cos(b) + sen(b). cos(a ) 7) sen(a − b) = sen(a). cos(b) − sen(b). cos(a ) 8) cos(a + b) = cos(a). cos(b) − sen(a). sen(b) 9) cos(a − b) = cos(a ). cos(b) + sen(a). sen(b)  π  p/ a ≠ + kπ 2 tg (a ) + tg (b)   π 10) tg ( a + b) =  p/ b ≠ + kπ 1 − tg (a).tg (b)  2 p/ (a + b) ≠ π + kπ   2  π  p/ a ≠ + kπ 2 tg (a) − tg (b)   π 11) tg ( a − b) =  p/ b ≠ + kπ 1 + tg (a).tg (b)  2 p/ (a − b) ≠ π + kπ   2 As fórmulas acima são verdadeiras para arcos positivos, cuja soma pertence ao primeiro quadrante. 41
  • 42. Fórmulas da Multiplicação 12) sen(2 x) = 2. sen( x). cos( x) 13) cos(2 x) = cos 2 ( x) − sen 2 ( x) 2.tg ( x) 14) tg (2 x) = 1 − tg 2 ( x) Fórmulas da Transformação em Produto x+ y x− y 15) sen( x) + sen( y ) = 2. sen  . cos   2   2  x− y x+ y 16) sen(x) - sen(y) = 2. sen . cos   2   2  x+ y x− y 17) cos( x) + cos( y ) = 2. cos . cos   2   2  x+ y x− y 18) cos( x) − cos( y ) = −2. sen . sen   2   2  42
  • 43. FUNÇÃO DE 1º GRAU Definição Chama-se função polinomial do 1º grau, ou função afim, a qualquer função f de IR em IR dada por uma lei da forma f(x) = ax + b, onde a e b são números reais dados e a 0. Na função f(x) = ax + b, o número a é chamado de coeficiente de x e o número b é chamado termo constante. Veja alguns exemplos de funções polinomiais do 1º grau: f(x) = 5x - 3, onde a = 5 e b = - 3 f(x) = -2x - 7, onde a = -2 e b = - 7 f(x) = 11x, onde a = 11 e b = 0 Gráfico O gráfico de uma função polinomial do 1º grau, y = ax + b, com a 0, é uma reta oblíqua aos eixos Ox e Oy. Exemplo: Vamos construir o gráfico da função y = 3x - 1: Como o gráfico é uma reta, basta obter dois de seus pontos e ligá-los com o auxílio de uma régua: a) Para x = 0, temos y = 3 · 0 - 1 = -1; portanto, um ponto é (0, -1). b) Para y = 0, temos 0 = 3x - 1; portanto, e outro ponto é . Marcamos os pontos (0, -1) e no plano cartesiano e ligamos os dois com uma reta. x y 0 -1 0 Já vimos que o gráfico da função afim y = ax + b é uma reta. O coeficiente de x, a, é chamado coeficiente angular da reta e, como veremos adiante, a está ligado à inclinação da reta em relação ao eixo Ox. O termo constante, b, é chamado coeficiente linear da reta. Para x = 0, temos y = a · 0 + b = b. Assim, o coeficiente linear é a ordenada do ponto em que a reta corta o eixo Oy. 43
  • 44. Zero e Equação do 1º Grau Chama-se zero ou raiz da função polinomial do 1º grau f(x) = ax + b, a 0, o número real x tal que f(x) = 0. Temos: f(x) = 0 ax + b = 0 Vejamos alguns exemplos: 1. Obtenção do zero da função f(x) = 2x - 5: f(x) = 0 2x - 5 = 0 2. Cálculo da raiz da função g(x) = 3x + 6: g(x) = 0 3x + 6 = 0 x = -2 3. Cálculo da abscissa do ponto em que o gráfico de h(x) = -2x + 10 corta o eixo das abicissas: O ponto em que o gráfico corta o eixo dos x é aquele em que h(x) = 0; então: h(x) = 0 -2x + 10 = 0 x=5 Crescimento e decrescimento Consideremos a função do 1º grau y = 3x - 1. Vamos atribuir valores cada vez maiores a x e observar o que ocorre com y: 44
  • 45. x -3 -2 -1 0 1 2 3 y -10 -7 -4 -1 2 5 8 Notemos que, quando aumentos o valor de x, os correspondentes valores de y também aumentam. Dizemos, então que a função y = 3x - 1 é crescente. Observamos novamente seu gráfico: Regra geral: a função do 1º grau f(x) = ax + b é crescente quando o coeficiente de x é positivo (a > 0); a função do 1º grau f(x) = ax + b é decrescente quando o coeficiente de x é negativo (a < 0); Justificativa: • para a > 0: se x1 < x2, então ax1 < ax2. Daí, ax1 + b < ax2 + b, de onde vem f(x1) < f(x2). • para a < 0: se x1 < x2, então ax1 > ax2. Daí, ax1 + b > ax2 + b, de onde vem f(x1) > f(x2). Sinal Estudar o sinal de uma qualquer y = f(x) é determinar os valor de x para os quais y é positivo, os valores de x para os quais y é zero e os valores de x para os quais y é negativo. Consideremos uma função afim y = f(x) = ax + b vamos estudar seu sinal. Já vimos que essa função se anula pra raiz . Há dois casos possíveis: 45
  • 46. 1º) a > 0 (a função é crescente) y>0 ax + b > 0 x> y<0 ax + b < 0 x< Conclusão: y é positivo para valores de x maiores que a raiz; y é negativo para valores de x menores que a raiz 2º) a < 0 (a função é decrescente) y>0 ax + b > 0 x< y<0 ax + b < 0 x> Conclusão: y é positivo para valores de x menores que a raiz; y é negativo para valores de x maiores que a raiz. 46
  • 47. FUNÇÃO QUADRÁTICA Definição Chama-se função quadrática, ou função polinomial do 2º grau, qualquer função f de IR em IR dada por uma lei da forma f(x) = ax2 + bx + c, onde a, b e c são números reais e a 0. Vejamos alguns exemplos de função quadráticas: 1. f(x) = 3x2 - 4x + 1, onde a = 3, b = - 4 e c = 1 2. f(x) = x2 -1, onde a = 1, b = 0 e c = -1 3. f(x) = 2x2 + 3x + 5, onde a = 2, b = 3 e c = 5 4. f(x) = - x2 + 8x, onde a = 1, b = 8 e c = 0 5. f(x) = -4x2, onde a = - 4, b = 0 e c = 0 Gráfico O gráfico de uma função polinomial do 2º grau, y = ax2 + bx + c, com a 0, é uma curva chamada parábola. Exemplo: Vamos construir o gráfico da função y = x2 + x: Primeiro atribuímos a x alguns valores, depois calculamos o valor correspondente de y e, em seguida, ligamos os pontos assim obtidos. 47
  • 48. x y -3 6 -2 2 -1 0 0 0 1 2 2 6 Observação: Ao construir o gráfico de uma função quadrática y = ax2 + bx + c, notaremos sempre que: • se a > 0, a parábola tem a concavidade voltada para cima; • se a < 0, a parábola tem a concavidade voltada para baixo; Zero e Equação do 2º Grau Chama-se zeros ou raízes da função polinomial do 2º grau f(x) = ax2 + bx + c , a 0, os números reais x tais que f(x) = 0. Então as raízes da função f(x) = ax2 + bx + c são as soluções da equação do 2º grau ax2 + bx + c = 0, as quais são dadas pela chamada fórmula de Bhaskara: Temos: Observação A quantidade de raízes reais de uma função quadrática depende do valor obtido para o radicando , chamado discriminante, a saber: 48
  • 49. quando é positivo, há duas raízes reais e distintas; • quando é zero, há só uma raiz real; • quando é negativo, não há raiz real. Função Quadrática Coordenadas do vértice da parábola Quando a > 0, a parábola tem concavidade voltada para cima e um ponto de mínimo V; quando a < 0, a parábola tem concavidade voltada para baixo e um ponto de máximo V. Em qualquer caso, as coordenadas de V são . Veja os gráficos: 49
  • 50. Imagem O conjunto-imagem Im da função y = ax2 + bx + c, a 0, é o conjunto dos valores que y pode assumir. Há duas possibilidades: 1ª - quando a > 0, 50
  • 51. a>0 2ª quando a < 0, a<0 Construção da Parábola É possível construir o gráfico de uma função do 2º grau sem montar a tabela de pares (x, y), mas seguindo apenas o roteiro de observação seguinte: 1. O valor do coeficiente a define a concavidade da parábola; 51
  • 52. 2. Os zeros definem os pontos em que a parábola intercepta o eixo dos x; 3. O vértice V indica o ponto de mínimo (se a > 0), ou máximo (se a< 0); 4. A reta que passa por V e é paralela ao eixo dos y é o eixo de simetria da parábola; 5. Para x = 0 , temos y = a · 02 + b · 0 + c = c; então (0, c) é o ponto em que a parábola corta o eixo dos y. Sinal Consideramos uma função quadrática y = f(x) = ax2 + bx + c e determinemos os valores de x para os quais y é negativo e os valores de x para os quais y é positivos. Conforme o sinal do discriminante = b2 - 4ac, podemos ocorrer os seguintes casos: 1º - > 0 Nesse caso a função quadrática admite dois zeros reais distintos (x1 x2). a parábola intercepta o eixo Ox em dois pontos e o sinal da função é o indicado nos gráficos abaixo: quando a > 0 quando a < 0 y>0 (x < x1 ou x > x2) y>0 x1 < x < x2 y<0 x1 < x < x2 y<0 (x < x1 ou x > x2) 2º - =0 52
  • 53. quando a > 0 quando a < 0 3º - <0 quando a > 0 quando a < 0 EQUAÇÕES EXPONENCIAIS 53
  • 54. Chamamos de equações exponenciais toda equação na qual a incógnita aparece em expoente. Exemplos de equações exponenciais: 1) 3x =81 (a solução é x=4) 2) 2x-5=16 (a solução é x=9) 3) 16x-42x-1-10=22x-1 (a solução é x=1) 4) 32x-1-3x-3x-1+1=0 (as soluções são x’=0 e x’’=1) Para resolver equações exponenciais, devemos realizar dois passos importantes: 1º) redução dos dois membros da equação a potências de mesma base; 2º) aplicação da propriedade: a m = a n ⇒ m = n (a ≠ 1 e a > 0) EXERCÍCIOS RESOLVIDOS: 1) 3x=81 Resolução: Como 81=34, podemos escrever 3x = 34 E daí, x=4. 2) 9x = 1 Resolução: 9x = 1 ⇒ 9x = 90 ; logo x=0. x 3 81 3)   = 4 256 x x x 4 3 81 3 34 3 3 Resolução :   = ⇒   = 4 ⇒   =   ; então x = 4. 4 256 4 4 4 4 4) 3 x = 4 27 3 3 Resolução : 3 = 27 ⇒ 3 = 3 ⇒ 3 = 3 ; logo x = x 4 x 4 3 x 4 4 5) 23x-1 = 322x Resolução: 23x-1 = 322x ⇒ 23x-1 = (25)2x ⇒ 23x-1 = 210x ; daí 3x-1=10, de onde x=-1/7. 6) Resolva a equação 32x–6.3x–27=0. 54
  • 55. Resolução: vamos resolver esta equação através de uma transformação: 32x–6.3x–27=0 ⇒ (3x)2-6.3x–27=0 Fazendo 3x=y, obtemos: y2-6y–27=0 ; aplicando Bhaskara encontramos ⇒ y’=-3 e y’’=9 Para achar o x, devemos voltar os valores para a equação auxiliar 3x=y: y’=-3 ⇒ 3x’ = -3 ⇒ não existe x’, pois potência de base positiva é positiva y’’=9 ⇒ 3x’’ = 9 ⇒ 3x’’ = 32 ⇒ x’’=2 Portanto a solução é x=2 FUNÇÃO EXPONENCIAL Chamamos de funções exponenciais aquelas nas quais temos a variável aparecendo em expoente. A função f:IRIR+ definida por f(x)=ax, com a ∈ IR+ e a≠1, é chamada função exponencial de base a. O domínio dessa função é o conjunto IR (reais) e o contradomínio é IR+ (reais positivos, maiores que zero). GRÁFICO CARTESIANO DA FUNÇÃO EXPONENCIAL Temos 2 casos a considerar:  quando a>1;  quando 0<a<1. Acompanhe os exemplos seguintes: 1) y=2x (nesse caso, a=2, logo a>1) Atribuindo alguns valores a x e calculando os correspondentes valores de y, obtemos a tabela e o gráfico abaixo: x -2 -1 0 1 2 y 1/4 1/2 1 2 4 2) y=(1/2)x (nesse caso, a=1/2, logo 0<a<1) Atribuindo alguns valores a x e calculando os correspondentes valores de y, obtemos a tabela e o gráfico abaixo: 55
  • 56. x -2 -1 0 1 2 y 4 2 1 1/2 1/4 Nos dois exemplos, podemos observar que a) o gráfico nunca intercepta o eixo horizontal; a função não tem raízes; b) o gráfico corta o eixo vertical no ponto (0,1); c) os valores de y são sempre positivos (potência de base positiva é positiva), portanto o conjunto imagem é Im=IR+. Além disso, podemos estabelecer o seguinte: a>1 0<a<1 f(x) é crescente e Im=IR+ f(x) é decrescente e Im=IR+ Para quaisquer x1 e x2 do domínio: Para quaisquer x1 e x2 do domínio: x2>x1 ⇒ y2>y1 (as desigualdades têm x2>x1 ⇒ y2<y1 (as desigualdades têm mesmo sentido) sentidos diferentes) INEQUAÇÕES EXPONENCIAIS Chamamos de inequações exponenciais toda inequação na qual a incógnita aparece em expoente. 56
  • 57. Exemplos de inequações exponenciais: 1) 3 x > 81 (a solução é x > 4) 2 −1 2) 2 2x -2 ≤ 2 x (que é satisfeita para todo x real) x −3 4 4 3)   ≥   (que é satisfeita para x ≤ -3) 5 5 4) 25 x - 150.5 x + 3125 < 0 (que é satisfeita para 2 < x < 3) Para resolver inequações exponenciais, devemos realizar dois passos importantes: 1º) redução dos dois membros da inequação a potências de mesma base; 2º) aplicação da propriedade: a>1 0<a<1 a > a ⇒ m>n m n a > an ⇒ m<n m (as desigualdades têm mesmo sentido) (as desigualdades têm sentidos diferentes) EXERCÍCIO RESOLVIDO: − 11 1) 4 x −1 + 4 x − 4 x +1 > 4 Resolução : 4x − 11 A inequação pode ser escrita + 4 x − 4 x .4 > . 4 4 Multiplicando ambos os lados por 4 temos : 4 x + 4.4 x − 16.4 x > −11 , ou seja : (1 + 4 − 16).4 x > −11 ⇒ -11.4 x > −11 e daí, 4 x < 1 Porém, 4 x < 1 ⇒ 4 x < 4 0. Como a base (4) é maior que 1, obtemos : 4 x < 40 ⇒ x < 0 Portanto S = IR - (reais negativos) FUNÇÃO LOGARÍTMICA A função f:IR+IR definida por f(x)=logax, com a≠1 e a>0, é chamada função logarítmica de base a. O domínio dessa função é o conjunto IR+ (reais positivos, maiores que zero) e o contradomínio é IR (reais). 57
  • 58. GRÁFICO CARTESIANO DA FUNÇÃO LOGARÍTMICA Temos 2 casos a considerar:  quando a>1;  quando 0<a<1. Acompanhe nos exemplos seguintes, a construção do gráfico em cada caso: 3) y=log2x (nesse caso, a=2, logo a>1) Atribuindo alguns valores a x e calculando os correspondentes valores de y, obtemos a tabela e o gráfico abaixo: x 1/4 1/2 1 2 4 y -2 -1 0 1 2 4) y=log(1/2)x (nesse caso, a=1/2, logo 0<a<1) Atribuindo alguns valores a x e calculando os correspondentes valores de y, obtemos a tabela e o gráfico abaixo: x 1/4 1/2 1 2 4 y 2 1 0 -1 -2 58
  • 59. Nos dois exemplos, podemos observar que d) o gráfico nunca intercepta o eixo vertical; e) o gráfico corta o eixo horizontal no ponto (1,0). A raiz da função é x=1; f) y assume todos os valores reais, portanto o conjunto imagem é Im=IR. Além disso, podemos estabelecer o seguinte: a>1 0<a<1 f(x) é crescente e Im=IR f(x) é decrescente e Im=IR Para quaisquer x1 e x2 do domínio: Para quaisquer x1 e x2 do domínio: x2>x1 ⇒ y2>y1 (as desigualdades têm x2>x1 ⇒ y2<y1 (as desigualdades têm mesmo sentido) sentidos diferentes) 59
  • 60. EQUAÇÕES LOGARÍTMICAS Chamamos de equações logarítmicas toda equação que envolve logaritmos com a incógnita aparecendo no logaritmando, na base ou em ambos. Exemplos de equações logarítmicas: 7) log3x =5 (a solução é x=243) 2 8) log(x -1) = log 3 (as soluções são x’=-2 e x’’=2) 9) log2(x+3) + log2(x-3) = log27 (a solução é x=4) 2 10) logx+1(x -x)=2 (a solução é x=-1/3) Alguns exemplos resolvidos: 1)log3(x+5) = 2 Resolução: condição de existência: x+5>0 => x>-5 log3(x+5) = 2 => x+5 = 32 => x=9-5 => x=4 Como x=4 satisfaz a condição de existência, então o conjunto solução é S={4}. 2) log2(log4 x) = 1 Resolução: condição de existência: x>0 e log4x>0 log2(log4 x) = 1 ; sabemos que 1 = log2(2), então log2(log4x) = log2(2) => log4x = 2 => 42 = x => x=16 Como x=16 satisfaz as condições de existência, então o conjunto solução é S={16}. 3) Resolva o sistema: log x + log y = 7  3. log x − 2. log y = 1 Resolução: condições de existência: x>0 e y>0 Da primeira equação temos: log x+log y=7 => log y = 7-log x Substituindo log y na segunda equação temos: 3.log x – 2.(7-log x)=1 => 3.log x-14+2.log x = 1 => 5.log x = 15 => => log x =3 => x=103 Substituindo x= 103 em log y = 7-log x temos: log y = 7- log 103 => log y = 7-3 => log y =4 => y=104. Como essas raízes satisfazem as condições de existência, então o conjunto solução é S={(103;104)}. 60
  • 61. INEQUAÇÕES LOGARÍTMICAS Chamamos de inequações logarítmicas toda inequação que envolve logaritmos com a incógnita aparecendo no logaritmando, na base ou em ambos. Exemplos de inequações logarítmicas: 1) log2x > 0 (a solução é x>1) 2) log4(x+3) ≤ 1 (a solução é –3<x≤1) Para resolver inequações logarítmicas, devemos realizar dois passos importantes: 1º) redução dos dois membros da inequação a logaritmos de mesma base; 2º) aplicação da propriedade: a>1 0<a<1 logam > logan ⇒ m>n>0 logam > logan ⇒ 0<m<n (as desigualdades têm mesmo sentido) (as desigualdades têm sentidos diferentes) EXERCÍCIOS RESOLVIDOS: 1) log2(x+2) > log28 Resolução: Condições de existência: x+2>0, ou seja, x>-2 (S1) Como a base (2) é maior que 1, temos: x+2>8 e, daí, x>6 (S2) O conjunto solução é S= S1 ∩ S2 = {x ∈ IR| x>6}. Portanto a solução final é a intersecção de S1 e S2, como está representado logo abaixo no desenho: 2) log2(log3x) ≥ 0 Resolução: Condições de existência: x>0 e log3x>0 Como log21=0, a inequação pode ser escrita assim: log2(log3x) ≥ log21 Sendo a base (2) maior que 1, temos: log3x ≥ 1. Como log33 = 1, então, log3x ≥ log33 e, daí, x ≥ 3, porque a base (3) é maior que 1. As condições de existência estão satisfeitas, portanto S={x ∈ IR| x ≥ 3}. 61
  • 62. FUNÇÃO MODULAR • Módulo (ou valor absoluto) de um número O módulo (ou valor absoluto) de um número real x, que se indica por | x | é definido da seguinte maneira:  x, se x ≥ 0 x = − x, se x < 0 Então:  se x é positivo ou zero, | x | é igual ao próprio x. Exemplos: | 2 | = 2 ; | 1/2 | = | 1/2 | ; | 15 | = 15  se x é negativo, | x | é igual a -x. Exemplos: | -2 | = -(-2) = 2 ; | -20 | = -(-20) = 20 O módulo de um número real é sempre positivo ou nulo. O módulo de um número real nunca é negativo. Representando geometricamente, o módulo de um número real x é igual a distância do ponto que representa, na reta real, o número x ao ponto 0 de origem. Assim: • Se | x | < a (com a>0) significa que a distância entre x e a origem é menor que a, isto é, x deve estar entre –a e a, ou seja, | x | < a ⇔ -a < x < a. • Se | x | > a (com a>0) significa que a distância entre x e a origem é maior que a, isto é, deve estar à direita de a ou à esquerda de –a na reta real, ou seja: | x | > a ⇔ x > a ou x < -a. • Equações modulares Toda a equação que contiver a incógnita em um módulo num dos membros será chamada equação modular. Exemplos: a) | x2-5x | = 1 b) | x+8 | = | x2-3 | 62
  • 63. ALGUMAS EQUAÇÕES MODULARES RESOLVIDAS: 1) Resolver a equação | x2-5x | = 6. Resolução: Temos que analisar dois casos: caso 1: x2-5x = 6 caso 2: x2-5x = -6 Resolvendo o caso 1: x2-5x-6 = 0 => x’=6 e x’’=-1. Resolvendo o caso 2: x2-5x+6 = 0 => x’=3 e x’’=2. Resposta: S={-1,2,3,6} 2) Resolver a equação | x-6 | = | 3-2x |. Resolução: Temos que analisar dois casos: caso 1: x-6 = 3-2x caso 2: x-6 = -(3-2x) Resolvendo o caso 1: x-6 = 3-2x => x+2x = 3+6 => 3x=9 => x=3 Resolvendo o caso 2: x-6 = -(3-2x) => x-2x = -3+6 => -x=3 => x=-3 Resposta: S={-3,3} • Inequações modulares Chamamos de inequações modulares as inequações nos quais aparecem módulos de expressões que contém a incógnita. Algumas inequações modulares resolvidas: 1) Resolver a inequação | -2x+6 | < 2. Resolução:  − 2 < −2 x + 6 2 x < 6 + 2 | - 2x + 6 | < 2 ⇒ − 2 < −2 x + 6 < 2 ⇒  ⇒  ⇒ − 2 x + 6 < 2 − 2 x < 4 2 x < 8 x < 4 ⇒  ⇒  2 x > 4 x > 2 S = {x ∈ IR | 2<x<4} 63
  • 64. 2) Dê o conjunto solução da inequação |x2-2x+3| ≤ 4. Resolução: |x2-2x+3| ≤ 4 => -4 ≤ x2-2x+3 ≤ 4. Então temos duais inequações (que devem ser satisfeitas ao mesmo tempo): Eq.1: -4 ≤ x2-2x+3 Eq.2: x2-2x+3 ≤ 4 Resolvendo a Eq.1: -4 ≤ x2-2x+3 => -4-3 ≤ x2-2x => -7 ≤ x2-2x => x2-2x+7 ≥ 0 => sem raízes reais Resolvendo a Eq.2: x2-2x+3 ≤ 4 => x2-2x-1 ≤ 0  x' = 1 − 2  Aplicando Bhaskara encontramos as raízes   x' ' = 1 + 2  S = {x ∈ IR | 1 − 2 ≤ x ≤ 1 + 2} • Módulo e raiz quadrada Consideremos os números reais x e y. Temos por definição, que x=y se e somente se, y2 = x e y≥0. Daí podemos concluir que x2 = x só é verdadeiro se x≥0. Se tivermos x<0, não podemos afirmar que x2 = x pois isso contradiz a definição. Por exemplo, se x=-3, teríamos: ( −3) 2 = −3 o que é um absurdo, pois o primeiro membro é positivo e o segundo negativo. Usando a definição de módulo, podemos escrever: x 2 =| x | o que é verdadeiro para todo x real. Devemos proceder da mesma forma em relação a todas raízes de índice par: 4 x 4 =| x |, 6 x 6 =| x |, 2n x 2 n =| x |, com x ∈ IR e n ∈ IN * Com relação às raízes de índice ímpar, podemos escrever: 2 n +1 3 x 3 = x, 5 x 5 = x, x 2 n+1 = x, com x ∈ IR e n ∈ IN 64